Anda di halaman 1dari 92

rnftlnDg

KATA PENGANTAR

Dengan memanjatkan puji syukur ke Hadhirat Alloh SWT, buku “English


Grammar for Indonesian Students” ini penulis hadirkan kepada para pembaca
budiman, khususnya para pelajar dan mahasiswa yang sedang mempelajari Tata
Bahasa Inggris (English Grammar). Buku ini juga sangat membantu dan
bermanfaat bagi para guru bahasa Inggris di sekolah menengah dan para dosen
bahasa Inggris di perguruan tinggi.
“English Grammar for Indonesian Students” disajikan secara ringkas
dan mudah untuk dipahami oleh para pembelajar dan guru atau dosen bahasa
Inggris karena didisain secara sistematis yang terdiri dari penjelasan teori singkat
dalam bahasa Indonesia, contoh-contoh kalimat dalam bahasa Inggris, rumus-
rumus yang mudah untuk diingat, dan beberapa latihan serta kunci jawabannya.
Demikian mudah-mudahan buah tangan ini dapat bermanfaat bagi semua
pihak yang menggunakannya. Kritik, koreksi dan saran yang membangun sangat
diharapkan sekali oleh penulis. Sekian dan terima kasih.

Ciamis, 10 November 2016


Penulis

~0~
DAFTAR ISI

KATA PENGANTAR i
DAFTAR ISI ii
Unit 1 VERB TENSES 1
Unit 2 PRONOUNS 17
Unit 3 CONJUNCTIONS 25
Unit 4 CONDITIONAL SENTENCES 33
Unit 5 ELLIPTICAL CONSTRUCTIONS 42
Unit 6 DEGREE OF COMPARISON 49
Unit 7 NOUNS 58
Unit 8 CONCORD/AGREEMENT 68
Unit 9 MODAL AUXILIARIES 79
Unit 10 PASSIVE VOICE 87
REFERENSI 93
KUNCI JAWABAN 94

~1~
Unit 1 VERB TENSES
TEORI SINGKAT
1. Present Simple Tense

A. Pola kalimat:

Subject + be/Verb 1 (s/es) + object/ adjective/adverb

1. I/We/You/They work in the office/catch a butterfly.


2. He/She/Bill/Ann/It works in the factory/catches a bird.
3. I am/She is/We are happy every time.

B. Arti Umum:
Sesuatu yang terjadi terus menerus, selalu atau biasa terjadi.

C. Kegunaan:
Simple Present Tense melukiskan:
a. kejadian, kegiatan atau peristiwa yang berulang (custom) atau
kebiasaan (habitual action) yang dilakukan seseorang/sekelompok
orang. Penggunaan Tense ini biasanya dilengkapi dengan adverb of
frequency: always, often, usually, generally, sometimes, ocasionally,
seldom, never, hardly, dan rarely. Di samping itu Tense ini biasanya
dilengkapi dengan adverb of time: every day, every week, every
morning, every night, every …, once a week, twice a day, three times
a year, weekly, monthly, anually, misalnya:
 David always gets up early.
 I have breakfast at seven every morning.
b. Menyatakan kebenaran umum (general truth).
 The sun rises in the east.
 The sky is blue.
~2~
c. Menyatakan agenda harian, mingguan, tahunan, dan seterusnya.
 Moslems go to the mosque every day.
 My father goes to London once a month.
d. Kenyataan sekarang.
 Tom looks happy today.
 They feel sad every night.
e. Perintah atau suruhan.
 Drive your car carefully.
 Please come in and sit down.

2. Present Continuous Tense


A. Pola Kalimat:

Subject + be ( is/am/are) + V-ing + object/adverb

1. She/He is sleeping at home.


2. I am studying English.
3. You/We/They are listening to the radio.

B. Arti Umum:
Sesuatu yang terjadi atau berlangsung pada saat dibicarakan.

C. Kegunaan:
Present Continuous Tense digunakan untuk:
a. menunjukkan suatu kejadian, kegiatan atau peristiwa yang sedang
berlangsung ketika si penutur sedang berbicara. Tense ini biasanya
diiringi keterangan waktu (time signal) now dan at this moment,
misalnya:
 Fred is driving a car now.
 The government is developing our country at this moment.
b. Pengulangan suatu tindakan yang sama.
 They are often going abroad.
 He is often making a mistake.

~3~
c. Tindakan yang akan segera dilakukan terutama menyangkut kata kerja
gerak: go, come, do, walk, swim, run.
 She is going to have a baby.
 “I am coming, Mom”, Teddy shouted.
3. Present Perfect Tense
A. Pola Kalimat:

Subject + have/has + been/Verb-3 + object/ adjective

1. I/You/We/They have painted the house.


2. He/She/It has eaten some food in dining room.

B. Arti Umum:
Sesuatu yang terjadi pada waktu lampau dan masih ada hubungannya
dengan sekarang. Akibat-akibat kejadian itu masih terasa atau dapat
dirasakan atau masih ada sisa-sisa bekas kejadian yang belum lama terjadi.

C. Kegunaan:
Present Perfect Tense melukiskan:
a. Sesuatu yang telah sempurna dilakukan pada saat diucapkan atau
kejadian yang baru saja terjadi dan akibatnya masih dapat dirasakan
sampai sekarang. Kata keterangan yang sering disisipkan diantaranya:
just, already, once, yet dan not yet.
 We have just finished reading the famous novel.
 My sister has not arrived yet.
b. Menyatakan sesuatu pengalaman.
 My uncle has been in Australia.
 I have been in USA for five years.
c. Menyatakan suatu yang telah dimulai dari titik waktu atau selama
waktu tertentu dan masih berlanjut hingga sekarang. Biasanya ada
keterangan since dan for.
 Mr. Hartono has taught English since 1990.
 They have lived in Sydney for five years.

~4~
4. Present Perfect Continuous Tense
A. Pola Kalimat:

Subject + have/has + been + Verb-ing + object/adjective/adverb

1. I/You/We/They have been sleeping since ten until now.


2. He/She/It has been working in the workshop.

B. Arti Umum:
Sesuatu kejadian yang telah terjadi dan mulai dari masa lampau dan masih
terus berlangsung sampai saat ini.

C. Kegunaan:
Present Perfect Continuous Tense digunakan:
a. Untuk menyatakan suatu kejadian yang masih berlangsung sampai saat
ini (berawal dari satu titik waktu di masa lampau dan berlanjut terus
hingga sekarang). Biasanya disisipi keterangan since….atau since
…until/til atau for …..
 We have been studying English since we were at junior high.
 The baby has been sleeping since eight until now.
b. Untuk menyatakan suatu peristiwa yang belum lama terjadi.
 I have been walking all day today.
 Ann has been coming from the town.

5. Past SimpleTense
A. Pola Kalimat:

Subject + Verb-2 + object/adjective/adverb

1. I/You/We/They stopped the bus at the bus stop yesterday.


2. He/She/It wrote a letter last week.

~5~
3. I was/ He was/ They were sad when someone died yesterday.

B. Arti Umum:
Sesuatu/situasi atau satu kejadian yang terjadi pada titik waktu di masa
lampau.
C. Kegunaan:
Simple Past Tense digunakan:
a. Untuk menunjukkan suatu kegiatan atau kejadian yang dilakukan pada
waktu tertentu di masa lampau. Tense ini biasanya diiringi kata
keterangan waktu seperti: yesterday, last week, last month, last …, a
week ago, a month ago, … ago, dan seterusnya.
 My brother went to Singapore yesterday.
 They met one another a week ago.
b. Untuk menyatakan peristiwa yang hanya terjadi satu kali di masa yang
lampau.
 Bill and I were born in the same year, 1969.
 Indonesia was proclaimed on 17th August 1945.
c. Untuk menyatakan suatu kejadian, kegiatan atau peristiwa yang
dilakukan berulang-ulang di masa lampau.
 He worked hard when he was young.
 I always played tennis in the morning last year.

6. Past Continuous Tense


A. Pola Kalimat:

Subject + be (was/were) + Verb-ing + object/adjective/adverb

1. He/She/It was standing when I arrived there.


2. I/You/We/They were swimming while Andy was fishing.
3. This time in 1970, I was living in California.

B. Arti Umum:
Sesuatu/situasi yang sedang berlangsung pada waktu lampau.

~6~
C. Kegunaan:
Past Continuous Tense digunakan:
a. Untuk menunjukkan kegiatan, kejadian, atau peristiwa yang sedang
berlangsung di masa lampau ketika suatu kejadian, kegiatan atau
peristiwa lain terjadi.
 My brother was studying when I came home.
 While I was going to school yesterday, I saw her.
Tanda pada kondisi ini terdapat When + Subject + Verb-2 + Object,
Adjective, Adverb atau While + Subject + be +Verb-ing + O, A, etc.
b. Untuk menunjukkan kegiatan atau kejadian yang sedang terjadi di
masa lampau.
 We were watching television last night.
 I was reading a magazine yesterday morning.
 This time in 1969, they were living in the jungle.

7. Past Perfect Tense


A. Pola Kalimat

Subject + had + been/Verb-3 + object/adjective/adverb

1. He/She/It had come before I left for London.


2. I/You/We/They had forgiven him before he apologized.

B. Arti Umum:
Sesuatu yang sudah terjadi, sebelum kejadian lain terjadi pada waktu
lampau.

C. Kegunaan:
Past Perfect Tense digunakan:
a. Untuk menunjukkan suatu kegiatan atau kejadian yang sudah
dituntaskan di masa lampau sebelum suatu kegiatan atau peristiwa lain
terjadi.
 We had finished building the house before the World War II
broke out.

~7~
 After he had broken the window, he ran away.
b. Untuk menunjukkan suatu kegiatan yang telah selesai dalam waktu
tertentu di masa yang telah lalu.
 My grandfather had been a soldier for forty years.
 The painter had painted the house for two hours yesterday.
8. Past Perfect ContinuousTense
A. Pola Kalimat:

Subject + had + been + Verb-ing + object/adverb

1. He/She/It had been standing when it rained rapidly.


2. I/You/We/They had been studying when he went home.

B. Arti Umum:
Sesuatu yang kejadiaannya berlangsung sangat lama, dan masih ada
hubungannya dengan saat tertentu pada waktu lampau.

C. Kegunaan:
Past Perfect Continuous digunakan untuk:
a. Menunjukkan suatu kegiatan atau peristiwa yang masih berlangsung
atau terjadi dalam satu jangka waktu tertentu di masa lampau.
 I had been reading a book from 9.00 to 10.00 yesterday.
 He had been sleeping for five hours last night.
b. Membicarakan sesuatu yang kejadiaannya berlangsung sangat lama,
dan masih ada hubungannya dengan saat tertentu pada waktu lampau.
 Yesterday morning I got up and looked out of the window. The
sun was shining but the ground was very wet. It had been raining.
c. Membicarakan lama berlangsungnya dari sesuatu kejadian sebelum
sesuatu yang lain terjadi di masa lampau.
 The football match had to be stopped. They had been playing for half
an hour when there was a terrible storm.

9. Future SimpleTense

~8~
A. Pola Kalimat (1):

Subject + will/shall + be/Verb-1 + object/adverb

1. She/He will be alone tomorrow and her/his friend will leave her/him.
2. We/I shall go but our/my friend will come soon.

Pola Kalimat (2):

Subject + be(is, am, are) going to + Verb-1 + object/adverb

1. I am going to buy a new car next month.


2. He/She is going to go camping next week.
3. You/They/We are going to sail tomorrow night.

B. Arti Umum:
Sesuatu atau kegiatan yang akan dilakukan atau peristiwa yang mungkin
atau akan terjadi.

C. Kegunaan:
Future Simple Tense digunakan untuk:
a. Menunjukkan suatu kegiatan atau kejadian yang akan terjadi atau akan
dilakukan di masa yang akan datang. Tense ini biasanya dilengkapi
dengan keterangan waktu: tomorrow, next week, next month, next …,
tonight.
 He will come if he has more time.
 She is going to invite us next week.
 I shall do it soon.
 Are you going to meet her?
b. Membicarakan sesuatu yang akan dilakukan pada saat berbicara.
 I’m too tired to walk home. I think I will get a taxi.
c. Menawarkan sesuatu.
 That bag looks heavy. I will help you with it.
d. Menjanjikan sesuatu.
 Thank you for lending me the money. I will pay you back on
Sunday.
~9~
e. Menyetujui atau menolak sesuatu.
 I have asked John to help me but he will not.
f. Meminta seseorang untuk melakukan sesuatu.
 Will you shut the door, please?

10. Future Continuous Tense


A. Pola Kalimat:

Subject + will/shall + be/Verb-ing + object/adjective/adverb

1. I shall be studying English at ten o’clock tomorrow morning.


2. She will be swimming in the swimming pool tomorrow.

B. Arti Umum:
Suatu kejadianyang sedang berlangsung pada titik waktu tertentu di masa
yang akan datang.

C. Kegunaan:
Future Continuous Tense digunakan untuk menyatakan suatu kegiatan,
kejadian atau peristiwa yang sedang berlangsung atau terjadi pada titik
waktu tertentu di masa depan.
 This time next week I will be shopping in Singapore.
 By two o’clock next Sunday I will be playing golf.

11. Future Perfect Tense


A. Pola Kalimat:
Subject + will/shall + have + Verb-3 + object/adjective/adverb

1. I will have studied English in two hours.


2. You shall have gone by next year.

B. Arti Umum:
Suatu kejadian yang diperkirakan selesai atau tuntas di masa yang akan
datang.

C. Kegunaan:
~ 10 ~
Future Perfect Tense digunakan untuk menunjukkan suatu kejadian atau
kegiatan yang direncanakan telah tuntas atau dituntaskan di masa yang
akan datang. Kata keterangan yang digunakan misalnya: by then, by that
time, by next time, by next year, by nine o’clock, in five hours, in six years,
dan lain-lain.
 By the end of this month we shall have arrived in London.
 The Congress will have been over by two o’clock tomorrow.

12. Future Perfect Continuous Tense


A. Pola Kalimat:

Subject + will/shall + have + been + Verb-ing + object, adjective, adverb

1. On 2nd June 2004 She will have been working for five years.
2. I shall have been reading this novel by the end of this month.

B. Arti Umum:
Suatu kejadian yang diperkirakan masih berlangsung di masa yang akan
datang.

C. Kegunaan:
Future Perfect Continuous Tense digunakan untuk menyatakan suatu
kegiatan, kejadian atau peristiwa yang masih berlangsung dalam jangka
waktu tertentu di masa yang akan datang.
 On 1st August 2005 we shall have been in Australia for seven years.
 How long will you have been learning English by the end of this
semester.

~ 11 ~
Latihan Pilihan Ganda
1. Mr. Jack : What will you do in the factory tomorrow?
Mr. Bakri : I …… a new machine.
A. operated
B. operate
C. will operate
D. am operating
2. Rino : Where’s daddy, mummy?
Mother : He is in the living room. He ….. news on TV.
A. watched
B. watches
C. will watch
D. is watching
3. Tio : How do you go to school, Wan?
Wawan : By bicycle. But yesterday I … to school by bus because the tire of my bike got
flat.

A. go
B. went
C. will go
D. have gone
4. Lina : Did the boys study after class?
Rina : No.
Lina : What did they do?
Rina : They … football.
A. play
B. plays
C. played
D. will play
5. Afrilia : You look healthy now.
Aini : I … from my illness.
~ 12 ~
Afrilia : I’m glad to hear that. So we can study together again.
A. have recovered
B. am recovering
C. will recover
D. recovered
6. Daughter : Mom, all my friends have got their new dress now.
Mother : Don’t worry, dear. We … to the mall tomorrow.
Daughter : Oh, thank you Mom. I love you.
A. have gone
B. will go
C. went
D. go
7. Yessy : Where did you go last vacation?
Tommy : Nowhere. What about you?
Yessy : My classmates and I … camping in Cibubur.
Tommy : So you had a nice experience, didn’t you?
A. go
B. went
C. will go
D. are going
8. Nita : Tomorrow is Rina’s birthday party.
Will you go there?
Rini : Sure. My Mom … me a new gown.
It’s really nice. I want to wear it for the party.
A. will buy
B. has bought
C. wants to buy
D. is going to buy

~ 13 ~
Dialog Soal Nomor 9 dan 10.

Betty : What … (9) if there … (10) drought all over the world?
Marcella : A lot of hunger and death.

9. Pilihlah jawaban yang tepat.


A. will happen
B. has happened
C. happened
D. happens
10. Pilihlah jawaban yang tepat.
A. were
B. was
C. are
D. is

Latihan Melengkapi
1. Andrew and James sometimes … (have) lunch together.
2. I … (study) for an hour when you came last night.
3. At this time yesterday, my uncle … (wash) his car.
4. The class … (watch) the documentary film next Monday.
5. My little sister … (take) English course for three months by the end of this
year.
6. Who … (run) downstairs? It’s dangerous.
7. My mother is exhausted since she … (cook) in the kitchen all day.
8. … (complete) the scholarship application?
9. Brian … (not attend) my birthday party this night.
10. We had had our dinner when Jim … (buy) us some pizza.

~ 14 ~
Latihan Error Recognition
1. My sister work in a hospital and my little brother studies in a college.
A B C
2. Recently, the work had been doing by Mr Adams.
A B C
3. The thief who steal my watch ran to the hills.
A B C

4. By the end of this week, my mother arrives from Jakarta.


A B C
5. Who do you think would be the winner of the speech competition?
A B C
6. A desert area that has been without water for six years will still bloom
A B
when rain will come.
C
7. Peas requires rich soil and a cool growing season to develop well.
A B C
8. A rabbit moved about by hopping on its hind legs, which are much
A B
longer and stronger than its front legs.
C
9. A plumber fix your plumbs and kitchen sinks while a janitor cleans
A B
schools, offices, stores and other business places.
C
10. My mother and I go to Batik Center in Kampung Troso, Jepara
A B
last Sunday.
C

~ 15 ~
Unit 2 PRONOUNS
TEORI SINGKAT
Pronoun adalah kata ganti, baik untuk orang (person) maupun benda
(object, thing). Ada beberapa macam pronoun diantaranya:
A. Definite Pronoun (Kata ganti tentu)
1. Personal Pronoun (Kata ganti personal/perorangan)
Number Grade Subject Object
First person I me
Singular Second person you you
Third person he/she/it him/her/it
First person we us
Plural Second person you you
Third person they them

Contoh:
1. I see it.
2. He knows you.
3. They live there.
4. That’s him over there.
5. I saw her.
6. Tom likes them.
7. Bill found me a job, etc.
a. Degree (Tingkatan)
 The first (Pertama) : I, we
 The second (Kedua) : You
 The third (Ketiga) : He, she, it, they
b. Number (Jumlah)
 Singular (Tunggal) : I, you, he, she, it
 Plural (Jamak) : We, you, they
c. Gender (Jenis kelamin)
 Male (Pria) : He
 Female (Wanita) : She

~ 16 ~
 Neutral (Netral) : I, you, we, they, it
2. Possessive Pronoun (Kata ganti posesif/empunya)
Dependent Independent Dependent Independent
My Mine Its Its
Your Yours Our Ours
His His Their Theirs
Her Hers Your Yours

Contoh:
1. That is my book. That is mine.
2. I found your car in the park. Now yours is over there.
3. This is his house. This is his.
4. I gave her dolls. I gave hers last week.
5. I have a new bike. Its color is blue. Its is blue.
6. We have finished our programs. We did ours well.
7. They keep their children nicely. They keep theirs well.
3. Reflexive Pronoun (Kata ganti refleksif)
Possessive Reflexive Possessive Reflexive
My myself its itself
Your yourself our ourselves
Him himself them themselves
Her herself your yourselves

Catatan: ada kata ganti refleksif tak tentu (Indefinite Reflexive Pronoun) yaitu
oneself. One does his/her homework oneself (=Seseorang selalu mengerjakan
PR-nya sendiri). Refleksif ini bermakna ……. Sendiri (tanpa bantuan orang
lain)
Contoh:
1. I always do my homework myself.
(Saya selalu mengerjakan PR sendiri)
2. Do you often wash your clothes yourself?
(Apakah Anda sering mencuci pakaian sendiri?)
3. I told him not to lift the heavy box himself.
(Saya katakan padanya untuk tidak mengangkat sendiri kotak berat itu.)
4. I asked her to come herself.
(Saya meminta did datang sendiri.)
~ 17 ~
5. The cat carried out the fish itself.
(Kucing itu membawa ikan sendiri.)
6. We painted our houses ourselves.
(Kami mengecat sendiri rumah kami.) dan lain-lain.
4. Demontrative Pronoun (Kata ganti tunjuk)
Singular Plural Singular Plural
This These That Those

Contoh:
1. This hat is Tom’s.
2. These hats are for sale.
3. That is a horse.
4. Those horses are in the green field.

B. Indefinite Pronoun (Kata ganti tak tentu)


a. Each = setiap, diikuti kata kerja tunggal:
1) Each knows what to do.
2) Each comes on time.
b. Both = kedua-duanya, diikuti kata kerja jamak:
1) Both are open all day.
2) Both play in the school yard.
c. None = tak satupun/tak ada, bisa sebaga subyek atau obyek, diikuti kata
kerja tunggal:
1) We expected the letters, but none came.
2) I had some last year but I have none this year.
d. Another = yang lainnya (tunggal); others = yang lainnya (jamak):
1) Have you met David’s sisters? I’ve met one. I didn’t know he had
another.
2) Oh, he has two others.
e. Everything = segala sesuatu (sesuatu), diikuti kata kerja tunggal:
1) Everything has been wasted.
2) Everything comes true in my life.
f. Everybody/everyone = setiap orang (orang), diikuti kata kerja tunggal:

~ 18 ~
1) Everyone is ready.
2) Everybody recognizes him well.
g. Anything = sesuatu, digunakan dalam kalimat negatif atau bertanya:
1) There is not anything in the refrigerator.
2) Is there anything in your bag?
h. Anyone/anybody = seseorang, digunakan dalam kalimat negatif atau
bertanya:
1) I didn’t tell anyone/anybody about that.
2) Did anyone/anybody come to your house yesterday?
i. No one/nobody = tak seorang pun, diikuti kata kerja tunggal, digunakan
dalam kalimat positif:
1) No one can go to the wild jungle.
2) Nobody comes on time.
j. Nothing = tak ada (sesuatu), digunakan dalam kalimat positif:
1) I can do nothing.
2) There is nothing in the box.
k. Someone = seseorang, diikuti kata kerja tunggal, digunakan dalam kalimat
positif:
1) I met someone who knows me well.
2) Someone is knocking the door.

~ 19 ~
Latihan Pilihan Ganda
1. I can’t understand Carol when … speaks Spanish. It sounds weird. She should
study more.
A. they
B. she
C. he
D. yout
2. If you see my family next week, please give my best regards to .…
A. him
B. her
C. them
D. us
3. We can find various books in the library. ... is very complete.
A. It
B. Its
C. He
D. His
4. I can’t find … pencil. Can I borrow … George?
A. my, their
B. me, your
C. my, your
D. my, yours
5. This refrigerator is too expensive for … to buy. We need to think again.
A. them
B. her
C. us
D. it
6. James should control … words so that he wouldn’t say something bad.
A. her
B. his
C. their
D. our
~ 20 ~
7. The students were talking among … while Prof Jones was lecturing.
A. themselves
B. ourselves
C. yourselves
D. yourself
8. Frankly speaking, a woman often makes difficult of ….
A. myself
B. yourself
C. himself
D. herself
9. Hi, Brian! … are Michael and Tom, my nephews.
A. Those
B. These
C. That
D. This
10. Look at … man over there. Isn’t he familiar to you?
A. those
B. these
C. that
D. this

Latihan Melengkapi
1. Anita is my best friend. I know … very well.
2. Anita has a cat. … likes playing with ….
3. This is your pen, but I can’t find …. Do you see …?
4. Anton made this t-shirt ….
5. You can’t buy … with money.
6. … books belong to Raffa.
7. Somebody left … socks in the washing machine.
8. We helped … to some cola at the party.
9. They can invite … they want to their party.
10. Please listen to … Song. It’s very interesting.
~ 21 ~
Latihan Error Recognition

1. We wanted to arrive before dinner, but us flight was delayed.


A B C
2. This dictionary is you, and that one is mine.
A B C
3. My father is an architect. Him designed my house.
A B C
4. You should go to Jakarta youself to meet the client.
A B C
5. The students are discussing the lesson. Tell him that the time is over.
A B C
6. I have finished my project alone. I worked all night by meself.
A B C
7. This sandals are no longer fit for my feet.
A B C
8. Trademarks enable a company to distinguish its products from that of
A B C
another company.
9. As larvae, barnacles are free-swimming, but as adults they attach
A B
theirselves to stones, docks, and hulls of ships.
C
10. Is there everyone who agrees with feminism?
A B C

~ 22 ~
Unit 3 CONJUNCTIONS
TEORI SINGKAT
1. Correlative Conjunctions:
A. Both ... and …
(Both + noun/adverb + and + noun/adverb)
 Both my mother and my sister are here.
 The researcher project will take both time and money.
 Both cat and dog are pat which are kept as domestic animals.
 He writes both correctly and neatly.
B. Not only … but also ...
(Not only + noun/verb/adverb + but also + noun/verb/adverb)
 Not only my father but also my brother went to London
 Yesterday it not only rained but also snowed.
 He plays not only the piano but also the violin.
 She speaks English not only fast but also fluently.
C. Either ... or ...
(Either + noun + or + noun)
 He is good at either math or science.
 She is either rich or industrious.
 Either Peter or David is going to come to the general meeting.
 I will take either chemistry or physics next semester.
D. Neither ... nor ...
(Neither + noun/adjective + nor + noun/adjective)
 Neither she nor he passed the test.
 They speak neither English nor French.
 This program is neither important nor useful.
 That book is neither interesting nor accurate.
Catatan:
 Dua subjek yang dihubungkan dengan both …. and selalu
menggunakan kata kerja jamak. Sedangkan dua subjek yang dihubungkan dengan
~ 23 ~
not only …. but also, either …. or, atau neither …. nor, subjek yang paling
dekat dengan kata kerja biasanya berkata kerja tunggal atau jamak.
Contoh:
 Both David and Ann are students of English Education Program.
 Not only they but also we are going to see the manager.
 Not only he but also she is a state employee.
 Either Tom or Bill is accepted as a new student at Texas University.
 Either the Professor or the students are successful to do the experiments.
 Neither you nor I am happy to hear the bad news.
 Neither the parent nor the children are interested in looking at him.

2. Relationship Words:
A. Cause and Effect (Sebab Akibat)
1) Because = sebab, since = as = as long as = inasmuch = sebab/karena,
now that = dikarenakan sekarang atau saat ini.
Contoh:
 Because he was sleepy, he went to bed.
 Since he‘s not interested in classical music, he decided not to go to
the concert.
 Now that the semester is finished, I’m going to rest a few days and
then take a trip.
 As she had nothing in particular to do, she called up a friend and
asked her if she wanted to take in a movie
 As long as (So long as) you are not busy, could you help me with
this work?
 Inasmuch as the two government leaders could reach an
agreement, the possibilities for peace as still remote.
2) Therefore = oleh karena itu/oleh sebab itu, consequently =
akibatnya/maka dari itu.
Contoh:
 Fred didn’t study. Therefore, he failed the test.
 Fred didn’t study. Consequently, he failed the test.

~ 24 ~
3) So = therefore = as a result = maka dari itu, for = because = sebab
Contoh:
 He was tired, so he went to bed.
 The child hid behind his mother’s skirt, for he was afraid of the
dog.
4) Because of = due to = karena/dikarenakan/oleh karena.
Contoh:
 Because of the cold weather, we stayed home.
 Due to the cold weather, we stayed at home.

B. Opposition (Berlawanan)
1) Even though = although = though = walaupun/meskipun, whereas =
while = sedangkan/sementara.
Contoh:
 Even though it was cold, I went swimming.
 Although it was cold, I went swimming.
 Though it was cold, I went to swimming.
 Mary is rich, whereas John is poor.
 John is poor, while Mary is rich.
2) Nevertheless = nonetheless = however = akan tetapi/namun/tetapi.
Contoh:
 It was cold. Nevertheless, I went swimming.
 It was cold. Nonetheless, I went swimming.
 It was cold. However, I still went swimming.
3) But …anyway = but … still = yet … still = tetapi tetap saja.
Contoh:
 It was cold, but I went swimming anyway.
 It was cold, but I still went swimming.
 It was cold, yet I still went swimming.
4) Despite = in spite of = sekalipun/walaupun
 I went swimming despite the cold weather.
 I went swimming in spite of the cold weather.

~ 25 ~
C. Condition (Pengandaian)
1) If = jika, unless = if … not = jika tidak, only if = hanya jika, even if =
sekalipun/sungguhpun, whether or not = baik … maupun tidak ….,
provided (that) = providing (that) = jika/hanya jika, in case (that) = in
the event (that) = jika seandainya.
Contoh:
 If it rains, the streets get wet.
 I’ll go swimming tomorrow unless it’s cold.
 The picnic will be cancelled only if it rains
 Even if the weather is cold, I’m going to go swimming.
 I’m going to go swimming tomorrow whether or not it is cold.
 Providing/Provided (that) no one has any further questions, the
meeting will be adjourned.
 I’ll be at my uncle’s house in case (that) you (should) need to reach
me.
 In the event(that) you should need to reach me, I’ll be at my uncle’s
house.
2) Otherwise = kalau tidak.
Contoh:
 I always eat breakfast. Otherwise, I get hungry during class.
3) Or (else) = kalau tidak.
Contoh:
 I always eat breakfast, or (else) I get hungry during the class.
4) In case of = in the event of = jika seandainya.
Contoh:
 In case of trouble, call the police. = In case (that) there is a
trouble, call the police.
 In the event of rain, the picnic will be cancelled = In the event
(that) it rains, the picnic will be cancelled.

~ 26 ~
Latihan Pilihan Ganda
1. Whales, dolphins, and walruses are … fish … mammals.
A. not … but
B. not only … but also
C. either …… or
D. neither …….. nor
2. My mother has some shoes; .., none of them are comfortable for brisk walking.
A. however
B. otherwise
C. if
D. since
3. I lent my friend Rp. 100.000,- yesterday ... her wallet was lost.
A. or else
B. despite
C. if
D. since
4. … the woman … her daughter has a positive attitude.
A. but … and
B. not only … but also
C. either … or
D. neither ... nor
5. I would like to try … Rendang … Balado recipes.
A. both … and
B. not … but
C. either … or
D. neither ... nor
6. … being very rich, Andi never shows off.
A. Other than
B. Despite
C. Inspite
D. Because

~ 27 ~
7. … I feel well, I will go to the party.
A. however
B. otherwise
C. if
D. since
8. You mustn’t go out … the heavy rain.
A. although
B. otherwise
C. because of
D. because
9. Please come on time; …, we will miss the flight.
A. otherwise
B. therefore
C. so
D. because
10. Sinta worked very sincerely; … she was promoted.
A. otherwise
B. consequently
C. although
D. because

Latihan Melengkapi
1. The baby keeps the light on … he is afraid sleeping in the dark.
2. My mother is a vegetarian. She will not eat … beef … chicken.
3. Joni didn’t study last night, … he … passed the exam.
4. My brother was very tired; … he went to bed earlier.
5. You will not succeed … you work hard.
6. Justin Timberlake is … handsome … smart.
7. … Amat is not tall, he is a very good basketball player.
8. I’ll be in my room … you want to talk with me.
9. … Ani … Ana likes snakes; they are afraid of snakes.
10. I don’t care no more … you want to study.

~ 28 ~
Latihan Error Recognition
1. You’d better take an umbrella; although it rains.
A B C
2. He is poor; consequently, he is very happy.
A B C
3. Everyone is looking for him unless he is the only person who knows
A B
where the key is.
C
4. Please study hard, yet still you’ll fail the exam.
A B C
5. Unfortunately, not only my father nor my mother can attend my
A B
graduation ceremony.
C
6. Because I get good final scores, my father won’t buy me any bicycle.
A B C
7. His father died five years ago; otherwise all the responsibilities to
A B
raise the children fell on his mother.
C
8. He visited not only his grandmother and his uncle last week.
A B C
9. The Sony cam recorder demo was fun and impressive, in case it is
A B
not as good as Microsoft's.
C
10. Either breakfast nor lunch is included in the price.
A B C

~ 29 ~
Unit 4 CONDITIONAL SENTENCES

TEORI SINGKAT
If Clause atau Conditional Sentence adalah bentuk kalimat pengandaian
yang menggunakan IF sebagai kata penghubung. Makna IF CLAUSE dan
RESULT CLAUSE adalah:
1. Jika …, maka …. (If …, will/would/would have ….)
2. … akan … jika … ( … will/would/would have … if ….)
A. Tiga jenis “If Clause/Conditional Sentence”
1. Type I : Real Condition (True in the Present/Future)
(Sesuatu yang mungkin terlaksana/terjadi saat ini atau akan datang).

Result Clause IF Clause


Simple Future/Simple Present Simple Present
S + will + V1 +O/A/C
if + S + V1 + O/A/C
S + V1 + O/A/C
 I will go to school if it does not rain.
 I write to my parents every if I have enough time.
week
IF Clause Result Clause
Simple Present Simple Future/Simple Present
S + will + V1 +O/A/C
If + S + V1 + O/A/C,
S + V1 + O/A/C
 If it does not rain, I will go to school.
 If I have enough time, I write to my parents every week.

Contoh:
 If I don’t eat breakfast, I always get hungry during class.
 Water freezes (will freeze) if the temperature goes below 320F/00C.
 If the weather is nice tomorrow, we will go on a picnic.

2. Type II: Unreal Condition Present (Untrue in the Present/Future)


(Sesuatu yang bertentangan dengan kenyataan saat ini/mustahil terjadi
sekarang).

~ 30 ~
Result Clause IF Clause
Would + simple form Simple Past
S + would + V1 +O/A/C if + S + V2 + O/A/C
 I would go to school if it did not rain.
 I would write to my parents. if I had enough time now.
IF Clause Result Clause
Simple Past Would + simple form
If + S + V2 + O/A/C, S + would + V1 +O/A/C
 If it did not rain, I would go to school.
 If I had enough time now, I would write to my parents.

Contoh:
 If I had enough time now, I would write to my parents. (=In truth, I do
not have enough time, so I will not write to them.
 If I taught this class, I wouldn’t give tests.(=In truth, I don’t teach this
class.)
 If I were you, I would accept their invitation. (=In truth, I am not you.)
Catatan:
Were digunakan untuk semua subjek baik tunggal maupun jamak. Untuk
pemakaian informal, tetapi tidak umum digunakan secara gramatikal, was
digunakan untuk subjek I, he, she, it.

3. Type III: Unreal Condition Past (Untrue in the Past)


(Sesuatu yang bertentangan dengan apa yang telah terjadi di masa lampau):

Result Clause IF Clause


Would + have + V3 Past Perfect
S + would + have + V3 + O/A if + S + had + V3 + O/A
 I would have gone to school if it had not rained.
 I would have written to my if I had had enough time.
parents yesterday
IF Clause Result Clause
Past Perfect Would + have + V3
If + S + had + V3 + O/A, S + would + have + V3 + O/A
 If it had not rained, I would have gone to school.
 If I had had enough time, I would have written to my parents
yesterday.
Contoh:
 If you had told me about the problem, I would have helped you.(=In
truth, you did not tell me about it.)

~ 31 ~
 If they had studied, they would have passed the exam.(=In truth, they did
not study. They failed the exam.)
 If I hadn’t slipped on the ice, I wouldn’t been broken my arm.(=In truth, I
slipped on the ice. I broke my arm.)

B. Bentuk Inversi dari “Conditional Sentence” (Omitting “IF”)

Bentuk inversi dari Conditional Sentence atau If-nya dihilangkan akan membentuk pola

kalimat inversi sebagai berikut:

a. If I were you, I wouldn’t do that. Menjadi


Were I you, I wouldn’t do that.
b. If I had known, I would have told you. Menjadi
Had I known, I would have told you.
c. If anyone should call, please take a message. menjadi
Should anyone call, please take a message.
C. Menggunakan bentuk kata kerja “Progressive/V-ing”

TRUE CONDITIONAL
(Kondisi yang sebenarnya) (Bentuk pengandaiannya)
 It is raining now, so I will not go  If it were not raining right now, I
for a walk. would go for a walk.
 I am not living in Chile. I am not  If I were living in Chile, I would
working at a bank. be working at a bank.

D. Menggunakan paduan waktu “Mixed Time”

TRUE CONDITIONAL
(Kondisi yang sebenarnya) (Bentuk pengandaiannya)
 I did not eat breakfast several  If I had eaten breakfas several
hours ago, so I am hungry now. hours ago (past), I would not be
 He is not a good student. He did hungry. (present)
not study for the test yesterday.  If he were a good student
(present), he would have studied
for the test (past).

~ 32 ~
E. Menggunakan Modals : COULD, MIGHT, SHOULD

TRUE CONDITIONAL
(Kondisi yang sebenarnya) (Bentuk pengandaiannya)
 If I were a bird, I could fly home.  Could fly = would be able to fly
 If I could sing as well as you, I  Could sing = were able to sing
would join the opera.
 If I had had enough money, I  Could have gone = would have
could have gone to Florida for been able to go
vacation.
 If I don’t get a scholarship, I  I might get = maybe I will get
might get a job instead of going
to graduate school next fall.
 If you were a better student, you  You might get = maybe you
might get better grades. would get
 If you had told me about your  I might have been = maybe I
problem, I might have been able would have been
to help you
 If John should call, tell him I’ll  If John should call
be back around five. pengandaiannya itu lebih
 If there should be another world diragukan daripada If John calls.
war, the continued existence of  If there should be
the human race would be in pengandaiannya itu lebih
jeopardy. diragukan daripada If there were.

F. IF NOT = UNLESS (Jika tidak)


 If I did not eat breakfast several  Unless I ate breakfast several
hours ago, I will be hungry now. hours ago, I will be hungry now.
 If he was not a good student, he  Unless he was a good student, he
would not study for the test. would not study for the test.
 If you had not told me about the  Unless you had told me about the
problem, I would not have helped problem, I would not have helped
you you.
 If they had not studied, they  Unless they had studied, they
would not have passed the exam. would not have passed the exam.
 If I hadn’t slipped on the ice, I  Unless I had slipped on the ice, I
wouldn’t been broken my arm. wouldn’t been broken my arm.

~ 33 ~
Latihan Pilihan Ganda
1. I’m sorry, Adam. I was sick last night; therefore, I didn’t come to your party.
A. If I wasn’t sick last night, I came to your party.
B. If I had been sick last night, I wouldn’t come to your party.
C. If I hadn’t been sick last night, I would come to your party.
D. If I hadn’t been sick last night, I would have come to your party.
2. The sun always rises in the east.
A. What would happen if the sun rises in the west?
B. What will happen if the sun rises in the west?
C. What would happen if the sun rose in the west?
D. What would have happened if the sun had risen in the west?
3. I cannot finish this work unless you … me.
A. helped
B. don’t help
C. help
D. didn’t help
4. If the man … carelessly, he would get an accident.
A. drives
B. didn’t drive
C. doesn’t drive
D. drove
5. She would have returned the book …
A. if you asked her
B. unless you asked her
C. if you had asked her
D. unless you had asked her
6. If you hurt him, I … something on you.
A. I would have done
B. I should have done
C. I might have done
D. I would do

~ 34 ~
7. If I pass my final test, what …?
A. you will buy for me
B. will you buy for me
C. would you buy for me
D. you would buy for me
8. I ... a taxi if I had known it was such a long way.
A. had taken
B. would have taken
C. would take
D. will take
9. We will miss the first train, unless we … early.
A. get up
B. got up
C. had got up
D. don’t get up
10. They ... a new car if they had more money.
A. had bought
B. would buy
C. will buy
D. would have bought

Latihan Melengkapi
1. If I finish the assignment before Saturday, I … (submit) it to my lecturer.
2. If I had seen the concert, I … (tell) you about it last night.
3. Had Juki not interfered in his brother’s marital problems, there … (be) peace
between them.
4. She would give you the money if she … (have) it.
5. They would call you immediately if they … (need) help.
6. Had my mother arrived at the sale early, she … (find) a better dress.
7. If you have enough time, please … (clean) your room before you go to
school.
8. They could go for a drive if today … (be) Sunday.
~ 35 ~
9. My cat will always sleep if it … (watch) drama on television.
10. They would understand it if you ... (explain) it to them more slowly.

Latihan Error Recognition


1. If I graduate from university with good marks, my father would buy
A B C
me a motorbike.
2. If she was my girlfriend, I would be the happiest man in the world.
A B C
3. He could have done the exam this morning if he studied last night.
A B C
4. Unless I did the homework, I will be out from the class.
A B C
5. I’d send you a postcard while I was on holiday if I had had your
A B C
address.
6. If she gave back my money, I will save it in the bank.
A B C
7. If I were you, I would have applied for the job.
A B C
8. His father would add his pocket money if he studies well.
A B C
9. We could have gone out if the weather hasn't been so bad.
A B C
10. If I have lots of money, I would have gone around the world.
A B C

~ 36 ~
ELLIPTICAL
Unit 5 CONSTRUCTIONS
TEORI SINGKAT
Elliptical Construction adalah bentukan kalimat elips/elipsis atau kalimat
sederhana. Elipsis merupakan bentuk penyederhanaan dua kalimat tunggal yang
memiliki unsur kalimat yang sama, (misalnya subjeknya, kata kerjanya atau
objeknya sama) menjadi kalimat majemuk.

1. Kalimat Positif
a. Pola kalimat (1)
Subject + Verb + and + Subject + Auxiliary + too

1) I go to school everyday and she does too.


2) She studied English and I did too.
3) We are happy now and they are too.
4) He must go as soon as possible and I must too.
5) You have arrived at school and they have too.

b. Pola kalimat (2)


Subject + Verb + and + so + Auxiliary + Subject

1) I go to school everyday and so does she.


2) She studied English and so did I.
3) We are happy now and so are they.
4) He must go as soon as possible and so must I.
5) You have arrived at school and so have they.

2. Kalimat Negative
a. Pola kalimat (1)
Subject + Negative Verb + and + Subject + Negative Auxiliary +
either

1) I do not go to school everyday and she doesn’t either.


2) She did not study French and I didn’t either.

~ 37 ~
3) We are not happy now and they aren’t either.
4) He cannot come on time and I can’t either.
5) You haven’t passed the exam and they haven’t either.

b. Pola kalimat (2)

Subject + Negative Verb + and + neither + Auxiliary + Subject

1) I do not go to school everyday and neither does she.


2) She did not study French and neither did I.
3) We are not happy now and neither are they.
4) He cannot come on time and neither can I.
5) You haven’t passed the exam and neither have they.

3. Kalimat Berlawanan
a. Pola kalimat (1)
Subject + Verb + but + Subject + Negative Auxiliary

1) I come to school on time but he doesn’t.


2) She can do the work well but they can’t.
3) He gets up early in the morning but you don’t.
4) We finished reading all chapters but she didn’t.
5) They will fight for the game but we won’t.

b. Pola kalimat (2)


Subject + Negative Auxiliary + Verb + but + Subject + Auxiliary

1) I do not come to school on time but he does.


2) She cannot do the work well but they can.
3) He does not get up early in the morning but you do.
4) We did not finish reading all chapters but she did.
5) They will not fight for the game but we will.

~ 38 ~
4. Pola kalimat elips lainya
a. Subjek berbeda; kata kerja sama
 You and I will be together next time.
b. Subjek sama; kata kerja berbeda
 They come and sit in front of the classroom.
c. Subjek dan objek sama; kata kerja berbeda
 We buy and read that book.

Latihan Pilihan Ganda


1. My little sister takes English course every Wednesday, but ....
A. my cousin does
B. my cousin did
C. my cousin doesn’t
D. my cousin didn’t
2. I will go to the traditional market tomorrow, and ....
A. Maria would too
B. Maria will too
C. Maria has too
D. Maria will go too
3. I cannot accompany my mother to the airport, and ....
A. my brother can’t either
B. my brother couldn’t either
C. my brother can’t neither
D. my brother couldn’t neither
4. I was buying some comics at the bookstore yesterday, and ....
A. so is my little sister
B. so has my little sister
C. so was my little sister
D. so were my little sister

~ 39 ~
5. Nada didn’t study last night. I didn’t study last night.
Nada didn’t study last night, and ….
A. neither did I
B. I did either
C. so did I
D. I did too
6. Krakatau Steel is located in Cilegon. Kratakatau Posco is located in Cilegon.
Krakatau Steel is located in Cilegon, and ….
A. so is Krakatau Posco
B. Krakatau Posco isn’t either
C. neither is Krakatau Posco
D. neither isn’t Krakatau Posco
7. Mahmud has a mustache. Mr. Nadjib has a mustache.
Mahmud has a mustache, and ….
A. neither does Mr. Nadjib
B. Mr. Nadjib does too
C. Mr. Nadjib doesn’t either
D. Mr. Nadjib has too
8. Siraj hasn’t seen “Thor” new movie. Fahmi hasn’t seen “Thor” new movie.
Siraj hasn’t seen “Thor” new movie, and ….
A. Fahmi has too
B. so has Fahmi
C. neither hasn’t Fahmi
D. Fahmi hasn’t either
9. I watched Fast and Furious 6 at the cinema yesterday, but ….
A. my friend doesn’t
B. my friend didn’t
C. my friend does
D. my friend did

~ 40 ~
10. Syifa won’t join us for dinner tonight. Divya won’t join us for dinner tonight.
Syifa won’t join us for dinner tonight, and ….
A. so will Divya
B. Divya won’t too
C. neither will Diva
D. Divya will either

Latihan Melengkapi
1. The students make beautiful carvings. Their teacher makes a beautiful carving.
The students make beautiful carvings, and ....
2. I don’t like smoking. He doesn’t like smoking.
I don’t like smoking, and ....
3. My brother bought a new television. My uncle bought a new television.
My brother bought a new television, and ....
4. He can’t play football. You can’t play football.
He can’t play football, and ....
5. Bob : I won't come to the meeting tomorrow. What about you?
Janet : Well, I'm afraid ....
6. His shop was burnt down, and … his car that was parked nearby.
7. I think that my mother will be interested in reading this novel,
but my father ….
8. They play football well. We play football well.
They play football well and ....
9. Doni likes ice cream. I don’t like ice cream.
Doni likes ice cream, while I ....
10. The boys are tired. The girls are tired.
The boys are tired and ....

~ 41 ~
Latihan Error Recognition
1. I started working at the publishing company tomorrow, and so will Ann.
A B C
2. I am not going to go shopping by myself and neither isn’t my mother.
A B C
3. My brother played tennis every weekend, whereas my sister doesn’t.
A B C
4. Sinta didn’t come to my birthday party, and Mike didn’t neither.
A B C
5. My mother will give me beautiful clothes as my birthday present, and
A B C
my aunt did too.

6. You have finished the homework and I had too.


A B C

7. My mother could not sing and I can’t either.


A B C
8. I were in Bali in 1992, and so were they.
A B C
9. Andi and Ana would study abroad, while Dito and Dina won’t.
A B C
10. Hadi doesn’t speak Italian, either does Hendra.
A B C

~ 42 ~
Unit 6 DEGREE OF COMPARISON

TEORI SINGKAT
Degree of Comparison adalah tingkatan-tingkatan perbandingan untuk kata
sifat (adjective) dan kata keterangan—yang bukan kata benda (adverb) yang
terdiri dari tiga tingkatan: 1) tingkat biasa (positive degree), 2) tingkat lebih
(comparative degree), dan 3) tingkat paling/ter-(superlative).

I. Degree of Comparison (Adjective):


A. Positive Degree (se-… /sama … dengan)
Digunakan untuk membandingkan dua buah benda yang memiliki sifat
yang sama. Kata sifat yang dibandingkan bersuku kata satu atau dua,
misalnya: big, easy, small, cheap, short, happy, long, tall, young, bad, far,
good, little, many, much, old; lebih dari dua suku kata, seperti: important,
beautiful, interesting, comfortable, handsome, difficult, effective,
attractive, expensive, dll.

Pola 1: as + adjective + as

Contoh:
a. My brother is a smart as your brother.
b. Your sister is as beautiful as my sister.

Pola 2: not as + adjective + as = not so + adjective + as

Contoh:
a. David is not as tall as Fred.
b. Fred is not so young as David.

B. Comparative Degree (lebih …dari)


Digunakan untuk membandingkan dua buah benda yang memiliki sifat yang berbeda.

Pola 1: adjective + -er + than

~ 43 ~
Pola ke-1 digunakan untuk kata sifat (adjective) yang terdiri dari satu atau
dua suku kata, baik yang beraturan, misalnya: big-bigger, easy-easier,
small-smaller, cheap-cheaper, short-shorter, happy-happier, long-longer,
tall-taller, young-younger, dll.; maupun yang tidak beraturan, seperti: bad-
worse, far-farther/further, good-better, little-less, many/much-more,
older(untuk orang & benda)/elder (utk orang).
Contoh:
a. His pencil is longer than mine.
b. It is better than never.

Pola 2: more + adjective + than

Pola ke-2 digunakan untuk kata sifat (adjective) yang beraturan dan
bersuku kata lebih dari dua, misalnya: more important, more beautiful,
more interesting, more comfortable, more handsome, more difficult, more
effective, more attractive, more expensive, dll.
Contoh:
a. Action is more important than talking.
b. She is more beautiful than her sister.

C. Superlative Degree (paling … /ter-…)


Digunakan untuk membandingkan lebih dari dua buah benda yang sifatnya
berbeda.

Pola 1: The + adjective + -est

Pola ke-1 digunakan untuk kata sifat (adjective) terdiri dari satu atau dua
suku kata, baik yang beraturan, misalnya: big-biggest, easy-easiest, small-
smallest, cheap-cheapest, short-shortest, happy-happiest, long-longest,
tall-tallest, young-youngest, dark-darkest, tallest, dll.; maupun yang tidak
beraturan, seperti: bad/ill-worst, far-farthest/furthest, good-best, little-
least, many/much-most, oldest (orang&benda)/eldest (orang).

~ 44 ~
Contoh:
a. Bill is the biggest boy in his class.
b. John is the tallest in our class.

Pola 2: The most + adjective

Pola ke-2 digunakan untuk kata sifat (adjective) yang beraturan dan
bersuku kata lebih dari dua, misalnya: most important, most beautiful,
most interesting, most comfortable, most difficult, most effective, most
attractive, most expensive, dll.
Contoh:
a. Aisyah is the most beautiful of those girls in my village.
b. The most interesting place for me in the world is Mecca.

II. Degree of Comparison (Adverb):


A. Positive Degree (se-…/sama …dengan)
Digunakan untuk membandingkan dua kegiatan yang dilakukan dengan derajat cara yang
sama.

Pola 1: as + adverb + as

Pola ke-1 digunakan untuk Adverb yang bersuku kata satu, dua atau lebih, baik yang
beraturan, mislanya: slowly, calmly, happily, nicely, sadly, easily, quickly, beautifully,
carefully, kindly, dll; maupun yang tidak beraturan, seperti: well, fast, hard, little, musch,
badly, far.

Contoh:
a. He works as slowly as he dared.
b. He did his job as well as I did

Pola 2: not as + adverb + as = not so + adverb + as

Contoh:
a. He does not snore as loudly as you do.
b. It did not take so long as I expected.

~ 45 ~
B. Comparative Degree (lebih …dari)
Digunakan untuk membandingkan dua kegiatan yang mempunyai cara derajatnya berbeda.

Pola 1: adverb +(-er) + than

Pola ke-1 digunakan untuk adverb yang tidak beraturan, yaitu: fast-faster,
hard-harder, well-better, little-less, early-earlier.
Contoh:
a. He played better than he had ever played.
b. They arrived earlier than I expected.

Pola 2: more + adverb + than

Pola ke-2 digunakan untuk adverb yang berakhiran –ly, misalnya:


beautifully, slowly, carefully, quickly, deeply, cleverly, carelessly,
interestingly, strongly, dan lain-lain.
Contoh:
a. She dances more beautifully than her sister does.
b. I run more slowly than you do.

C. Superlative Degree (paling …/ter-…)


Digunakan untuk membandingkan lebih dari dua buah kegiatan yang
derajatnya berbeda.

Pola 1: The + adverb + -est

Pola ke-1 digunakan untuk adverb berikut: well-best, badly-worst, little-least, much-most,
hard-hardest, early-earliest, far-farthest (distance)/furthest (used more widely).

Contoh:

a. Andy went furthest of the explorers.


b. David likes swimming best of all.

Pola 2: The most + adverb

~ 46 ~
Pola ke-2 digunakan untuk adverb yang berakhiran –ly, misalnya: most
quickly, most fortunately,
Contoh:
a. She behaved most generously.
b. He spoke most kindly of all men there.

Latihan Pilihan Ganda


1. Brazil is … England.
A. biggest than
B. bigger as
C. bigger than
D. biggest as
2. It was … expensive restaurant I've ever been to.
A. more
B. the more
C. most
D. the most
3. It's … art collection in Europe.
A. the finer
B. the finest
C. finer
D. finest
4. Ani : Do you prefer traveling by train or by plane?
Ita : I prefer traveling by train.
Ani : Why?
Ita : Buying the train ticket needs … money than the plane ticket.
A. fewer
B. few
C. less
D. little

~ 47 ~
5. Rio : Which is more interesting. The Safari Park or Ragunan Zoo?
Daud : I think Ragunan zoo is … as the Safari Park. Both of them
have many good things to see.
A. more interesting
B. less interesting
C. the most interesting
D. as interesting
6. This tie is too short for me. Show me a … one, please.
A. longer
B. more longer
C. longest
D. most longest
7. Excuse me, I can`t hear what you are talking about. Can you speak…?
A. louder
B. the loudest
C. more loudly
D. the most loudly
8. You got a poor grade in the second English test. You should study….
A. harder
B. the hardest
C. more hard
D. the most hard
9. The questions are not ... you think.
A. easy
B. as easy as
C. easier
D. easiest
10. I think your English is improving. It is getting ….
A. worst
B. worse
C. best
D. better

~ 48 ~
Latihan Melengkapi
1. Roger is 12 years old. Danu is 15 years old. Roger is … than Danu.
2. The elephant is the … animal on the earth.
3. A hill is … than a mountain.
4. This pencil is long. That pencil is short. That pencil isn’t as… as this pencil.
5. A horse runs … than a goat.
6. Almost all students think that Mathematic is the … subject in the school.
7. The scenery here is beautiful. But I think it will be … if there is no garbage anywhere.
8. Maya is the … student in my class. She never comes late to school.
9. An orangutan isn’t as … as an elephant. An elephant can lift a wood with its
trunk easily.
10. Arnold cleans his room twice a day. That’s why his room is the … among our
rooms.

Latihan Error Recognition


1. A negative number is little than zero.
A B C
2. My mother speaks English as more fluently as my sister.
A B C
3. Making the good decision for the organization is his role as a leader.
A B C
4. Living in San Fransisco is most expensive than living in Boston.
A B C
5. It was the usefulest thing I have ever learned.
A B C
6. Do women need much support after breaking bones?
A B C
7. This knife is as sharper as that one.
A B C
8. My father works the more hard in the water treatment company.
A B C
9. My brother works more hard than anybody else.
A B C
10. The patients were treated badly, but I was treated the badliest.
A B C

~ 49 ~
Unit 7 NOUNS
TEORI SINGKAT
Nouns adalah kata benda-kata benda yang memiliki fungsi tertentu dalam
suatu kalimat, misalnya sebagai subyek, obyek, atau keterangan subyek.
A. Kinds and Function of Nouns (Jenis dan Fungsi Kata Benda)
Dalam bahasa Inggris ada empat jenis kata benda, diantaranya:
1. Common Nouns
Common nouns adalah kata benda umum seperti: dog, man, table, house,
book, school, car, cat, dan lain-lain.
2. Proper Nouns
Proper nouns adalah kata benda nama diri, baik manusia, binatang, tumbuh-
tumbuhan, maupun benda mati, seperti: Indonesian, the Atlantic, the Thames,
the Alps, London, David, Marina, Mr. Smith, Mrs. Brown, dan lain-lain.
3. Abstract Nouns
Abstract nouns adalah kata benda abstrak yaitu kata benda yang tidak nyata,
seperti: beauty, charity, courage, fear, enjoy, happy, sad, dan lain-lain.
4. Collective Nouns
Collective nouns adalah kata benda kolektif, seperti: crowd, flock, group,
swarm, team, corps, family, staff, dan lain-lain.

B. Gender (Jenis Kelamin/Jender)


1. Masculine, Feminine & Neuter
Kata benda kelompok masculine: men, boys, father, uncle, grandfather,
nephew, son dan binatang jantan berkata ganti he/they. Kata benda
kelompok feminine: women, girls, mother, aunt, grandmother, niece,
daughter dan binatang betina berkata ganti she/they. Kata benda kelompok
neuter adalah benda mati dan benda hidup/binatang yang jenis kelaminnya
tidak diketahui, berkata ganti it/they, misalnya baby, child. Kendaraan
seperti cars, ships, countries dianggap feminine.
2. Masculine/feminine nouns denoting people
~ 50 ~
Kata benda maskulin atau feminin orang diantaranya:
a. boy-girls, bachelor-spinster, bridegroom-bride, gentleman-lady,
husband-wife, widow-widower.
Kekecualian (netral): baby, child, cousin, infant, parent, relation,
relative, spouse, teenager.
b. duke-duchess, earl-countess, king-queen, lord-lady, prince-princess.
c. Kata benda yang menunjukkan profesi/pekerjaan:
Artist, cook, assistant, dancer, driver, doctor, guide, etc. Kekecualian
bentuk pasangan gender/jenis kelaminnya: actor-actress, conductor-
cunductress, heir-heiress, hero-heroine, host-hostess, manager-
manageress, steward-stewardess, waiter-waitress, salesman-
saleswoman.

3. Domestic animals and many else:


Untuk binatang-binatang berikut ini mempunyai pasangan gender/jenis
kelaminnya:
Bull-cow, cock-hen, dog-bitch, duck-drake, gander-goose, lion-lioness,
ram-ewe, satg-doe, stallion-mare, tiger-tigress.

C. Singular & Plurals (Tunggal & Jamak)


1. Bentuk jamak kata benda biasanya dibuat dengan cara menambahkan
akhiran -s pada bentuk tunggalnya, misalnya: day-days, dog-dogs, house-
houses, etc. Akhiran -s yang diucapkan /s/ setelah kata benda yang berakhir
dengan suara p, k, f seperti desks berakhir dengan suara /s/ kecuali
diucapkannya /z/. Jika -s ditempatkan setelah -ce, -ge, -se, -ze suara suku
kata /iz/ ditambahkan, seperti: houses berakhir dengan suara /iz/.
2. Bentuk jamak dari kata benda-kata benda yang berakhiran -o, -ch, -sh, -ss,
-x adalah ditambah akhiran –es yang diucapkan /iz/: tomato-tomatoes,
touch-touches, sketch-sketches, brush-brushes, flash-flashes, kiss-kisses,
dress-dresses, box-boxes, tax-taxes, dan lain-lain. Tetapi untuk beberapa
kata pinjaman asli yang berakhiran –o, maka tambahkan saja akhiran –s:
dynamo-dynamos, kilo-kilos, kimono-kimonos, piano-pianos, photo-photos,

~ 51 ~
radio-radios, soprano-sopranos, memo-memos, auto-autos, solo-solos,
studio-studios, tatoo-tatoos, video-videos, zoo-zoos, dan lain-lain
(kekecualian: echo-echoes, hero-heroes, potato-potatoes, tomato-tomatoes;
mosquito-mosquitos/mosquitoes, tornado-tornados/tornadoes, volcano-
volcanos/volcanoes, zero-zeros/zeroes.)
3. Kata benda yang berakhir huruf y yang mengikuti huruf konsonan, bentuk
jamaknya dengan cara menghilangkan huruf y dan menambahkan akhiran -
ies: baby-babies, country-countries, fly-flies, lady-ladies. Tetapi jika huruf
akhir -y suatu kata benda itu mengikuti huruf vokal maka hanya dengan
menambahkan akhiran -s: boy-boys, day-days, donkey-donkeys, guy-guys.
4. Dua belas kata benda berakhiran -f atau -fe bentuk jamaknya dengan cara
menghilangkan -f atau -fe dan tambahkan -ves. These nouns are calf-
calves, half-halves, knife-knives, leaf-leaves, life-lives, loaf-loaves, self-
selves, sheaf-sheaves, shelf-shelves, thief-thieves, wife-wives, wolf-wolves.
Kecuali untuk hoof-hoofs/hooves, scarf-scarfs/scarves, wharf-
wharfs/wharveshoof-hoofs/hooves, scarf-scarfs/scarves, wharf-
wharfs/wharves tetapi cliff-cliffs, handkerchief-handkerchiefs, safe, safes.
5. Ada beberapa kata benda bentuk jamaknya dengan cara perubahan huruf
vokal: foot-feet, louse-lice, mouse-mice, woman-women, goose-geese, man-
men, tooth-teeth.
6. Untuk nama-nama makhluk hidup tertentu tidak mengalami perubahan
bentuk jamak, seperti beberapa jenis ikan (fish) secara normal tidak
berubah: ficarp, fish, cod, mackerel, pike, plaice, salmo, trout, tubot,
squid. Sekalipun ada bentuk jamak fishes dari fish tetapi tidak umum.
Tetapi jika maknanya jamak, hanya ditambahkan -s: crabs, eels, lobster,
sharks, sardines, herrings. Untuk jenis binatang lain ikan juga tidak
mengalami perubahan: deer, sheep. Bagi para pemburu, untuk jenis
binatang berikut: duck, patridge, pheasant, game=animal(s) tidak
mengalami perubahan bentuk jamak. Juga untuk kata benda-kata benda
berikut: aircraft, craft (boat/boats), quid, counsel.

~ 52 ~
7. Untuk kata benda kolektif: crew, family, team, jury, dsb., dapat bermakna
tunggal atau jamak tanpa merubah bentuknya. Contoh: Our team is the
best. Our team are wearing their new jerseys.
8. Kata benda-kata benda berikut selalu bermakna jamak walaupun bentuknya
tunggal, segala macam pakaian , alat dan perkakas yang mempunyai dua
bagian atau pasangan: clothes, police, breeches, pants, pyjamas, trousers,
binoculars, pliers, scissors, spectacles, glasses, scales, shears, dan lain-
lain.
9. Untuk beberapa istilah ilmu pengetahuan yang berakhiran (-ics) yang
berbentuk jamak dianggap tunggal: acoustics, ethics, hysterics,
mathematics, physics, politics, dan lain-lain. Sehingga kata kerjanya
berbentuk tunggal: mathematics is an exact science. Kecuali untuk jenis
kalimat: Her mathematics are weak (bentuk jamak).
10. Ada beberapa kata benda berbentuk jamak bermakna tunggal: news,
mumps, rickets, shingles, billiards, darts, dominoes.
11. Beberapa kata yang berbentuk asli Latin memiliki aturan perubahan bentuk
tunggal ke bentuk jamaknya: criterion-criteria, datum-data, phenomenon-
phenomena, cactus-cacti/cactuses, stimulus-stimuli, syllabus-
syllabi/syllabuses, formula-formulae/formulas, vertebra-vertebrae,
analysis-analyses, basis-bases, crisis-crises, hypothesis-hypotheses, oasis-
oases, parenthesis-parentheses, thesis-theses, appendix-
appendices/appendixes, index-indices/indexes, bacterium-bacteria,
curriculum-curricula, medium-media, erratum-errata, memorandum-
memoranda, radius-radii, terminus-termini, libretto-libretti, tempo-tempi.
Tetapi mungkin juga: librettos, tempos. Ada kata-kata latin yang
mengikuti pola perubahan bentuk jamak bahasa Inggris: dogma-dogmas,
formula-formulas/formulae.

D. Compound Nouns (Kata Benda Majemuk)


1. Pada umumnya kata terakhir berbentuk jamak: boy-friends, break-ins,
travels tetapi tidak untuk: men-drivers, women-drivers.

~ 53 ~
2. Kata benda pertama dalam kata benda majemuk dibuat bentuk tunggal jika
pola frase kata bendanya noun (verb+ er) + adverb: hangers-on, lookers-
on, runners-up. Pola noun + preposition + noun: ladies-in-waiting, sisters-
in-law, wards of courts.

E. Uncountable Nouns (Kata Benda Tak Terhitung)


1. Nama-nama zat umum: bread, beer, cloth, coffee, cream, dust, gin, glass,
gold, ice, jam, oil, paper, sand, soap, stone, tea, water, wine, wood.
2. Kata benda abstrak: advice, beauty, courage, death, experience, fear, help,
hope, horror, information, knowledge, mercy, pity, relief, suspension, work.
3. Kata benda-kata benda berikut dianggap Uncountale Nouns dalam bahasa
Inggris: baggage, camping, damage, furniture, luggage, parking, shopping,
weather.
Uncountable nouns selalu menggunakan persesuaian dengan kata kerja
tunggal: Her hair is black.
4. Uncountable nouns dapat dipadukan dengan satuan jumlah: a bit of news, a
grain of sand, a pail of paint, a drop of oil, a cake of soap, a pane of glass,
a piece of advice, a pot of jam, a sheet of paper.

Latihan Pilihan Ganda


1. I wonder how to climb trees without ….
A. branch
B. branchs
C. branches
D. branchez
2. Stamped kitchen … are usually cheaper than forged kitchen ….
A. knife
B. knifes
C. knive
D. knives

~ 54 ~
3. My mother cleaned up her sink after finding two … there.
A. mouse
B. mice
C. mouses
D. mices
4. Each of the participants of seminar … given a book and a briefcase.
A. he is
B. is
C. are
D. they are
5. Most of the government officials ... turn in their papers before the due date.
A. have to
B. has to
C. having to
D. is having to
6. My brother has … of playing cards.
A. a box
B. a slice
C. a pack
D. a glass
7. My mother bought …. of bananas.
A. a bunch
B. a group
C. a pack
D. a herd
8. The company has some ... in recruiting its employers.
A. criterion
B. criteria
C. criterions
D. criterias

~ 55 ~
9. Hanna behaved badly yesterday; she needed to have some ... from her mother.
A. suggestion
B. suggestions
C. advice
D. advices
10. My little brother always drinks … of milk every morning.
A. a plate
B. a pan
C. a bowl
D. a glass

Latihan Melengkapi
Change the words in the brackets with correct plural nouns.
1. Hendra joined several … (community) in his school.
2. That hunter killed six … (deer) last week.
3. There are many … (thief) in big cities.
4. … (ox) and … (goose) are animals.
5. Scientists are always studying the … (phenomenon) of nature.
Change the words in the brackets with correct nouns.
6. Yesterday, Kiki went to the English … (depart) in the University to follow a
TOEFL test.
7. My father needs a full time rest during his … (recover).
8. The … (associate) of football in my regency celebrate its 16th Anniversary.
9. Can you show me the … (enter) of that building?
10. A … (happy) that you get from your family will last forever.

~ 56 ~
Latihan Error Recognition
1. I have so many plan. I am afraid that I can’t realize them.
A B C
2. He should control his word so that he wouldn’t say something bad.
A B C
3. I need help to finish my works. I am so busy these days and I am not
A B C
feeling well now.
4. Nana felt worried about the childs, so she asked her husband not to go
A B C
abroad for the job.
5. Money is not everything, the most important one is a happy where we
A B C
can be with our lovely family and friend around us.
6. I can’t understand their when they speak Spain. They seem weird.
A B C
They should study more.
7. There is a job for you. You must apply in this company; you must
A B
make a written job apply.
C
8. This company’s market is good enough, the products are always sold
A B
and many consumers need it.
C
9. Hans, our class leader is in the teacher office, there can be an
A
announce about our trip to Jogjakarta next holiday.
B C
10. If you to see my family next weekend, give them my best regards and
A B
don’t forget to bring the give.
C
~ 57 ~
Unit 8 CONCORD/AGREEMENT

TEORI SINGKAT
Concord/Agreement adalah persesuaian kaidah tata bahasa. Concord atau
agreement atau congruence atau correspondence adalah persesuaian formal dalam
persona (person), jenis (gender) atau tenses antara dua unsur atau lebih yang
menunjukkan tautan gramatik dalam kalimat.

A. Concord/Agreement terbagi dua yaitu:


1. Bilateral concord
adalah persesuaian antara dua unsur kalimat, misalnya antara finite verb
to be “am” dengan I. I dan am adalah pasangan yang tak dapat berubah.
2. Unilateral concord
adalah persesuaian antara satu unsur kalimat dengan beberapa unsur
seperti finite verb to be “are” bisa berpasangan dengan you, we dan they,
juga is dengan he dan she. Demikian pula I (you, we, they) go (do).
Bila hanya ada satu bentuk seperti auxiliary verbs “can, may, must, will”
dan sebagainya, maka di sana tidak ada persoalan Concord.

B. Beberapa macam Concord/Agreement:


1. Concord of Person
Persesuaian dalam pelaku (person) ditemukan antara subjek dengan finite
verb atau predicator.
a. Kata ganti orang ketiga tunggal dengan “finite verb present tense”

(-s/-es)

 He needs some money for buying his properties.


 She studies English twice a week in Oxford English Course.
b. Kata ganti orang pertama dan ketiga tunggal dengan “finite verb to be dan
to have”.
 I am/was a student of English Department of Colorado University.

~ 58 ~
 He/She is/was very proud of his/her success in the math competition.
 I have finished reading the books I borrowed from the library.
c. Kata ganti orang pertama tunggal atau jamak, kedua tunggal atau jamak,
dan ketiga jamak dengan “finite verb tanpa –s/-es.
 I go to school by bus every morning.
 We have breakfast at seven before going to work.
 You switch on the computer when you start to operate it.
 They come to school on time every day.
d. Kata ganti orang kedua tunggal atau jamak dan ketiga jamak dengan
“finite verb to be”.
 You are/were very kind to me.
 All of you are the member of English Students Association now.
 They are/were building their house.
e. Kata ganti orang pertama tunggal dan jamak, orang kedua tunggal atau
jamak dan orang ketiga jamak dengan “finite verb present tense to have.”
 I have been studying English since 1989 until now.
 We have come from London just now.
 You have just arrived at home, haven’t you?
 They have gone abroad.
f. Kata benda (noun) atau kata ganti (pronoun) dengan pronoun yang
mengacu padanya yang bersifat bilateral.
 We love our parents forever.
 Someone brings his/her bag on the right hand.
 I could have kicked myself.
 Birds were building their nests.
 David and his friends are in the wedding party.

2. Concord of Number
Persesuaian dalam jumlah (number) dapat ditemukan antara lain:
a. Subjek dengan finite verb (predicator)nya.
 The settlers build their houses.
 The children play football in the play ground.

~ 59 ~
Berikut ini ada beberapa kata benda jamak yang menghendaki kata kerja
tunggal. Walaupun bentuknya jamak, maka benda tersebut lebih terasa
mengacu kepada gagasan tunggal:
1) Nama-nama permainan yang berakhir dengan (-s): billiards,
dominoes, draughts.
 Billiards is not exclusively a men’s game.
 Draughts is entirely a game of mathematial calculation. Tetapi
 Cards were strictly forbidden.
2) Nama-nama disiplin ilmu yang berakhiran (-ics): economics, ethics,
mechanics, phonetics, physics.
 Economics was thought by Professor Bernard.
 What is ethics? Tetapi
 Gymnastics are now a regular feature of popular education.

3) Nama negara The United States.


 Is the United States imperialist? Tetapi
 Nederlands are washed by the North Sea.
4) Kata benda works (=factory) mendapat finite verb tunggal.
 Price’s works was small.
 Smith’s works needs many employees.
5) Nama judul buku dalam bentuk jamak mendapat kata jerja tunggal.
 Perey’s Reliques was published in 1765.
 Albert’s Adventures was published in 1900.
6) Kata benda jamak sebagai kata itu sendiri.
 The Nederlands is the name of the country.
 Sumatrans is a group of people living in the Sumatra islands.
7) Kata benda yang mengacu kepada waktu, ukuran, dan sebagainya itu
didahului oleh numeral (angka), terutama bila ada predicative noun
yang menyatakan gagasan yang sama.
 Fifty years is along time.
 Twice ten is twenty.

~ 60 ~
b. Noun atau pronoun dengan yang kata ganti yang mengacu padanya.
 Students (they) have completed and submitted their papers to their
profesoor.
 Andy (he) is washing his car in front of his house.
c. Noun dengan keterangan atributifnya (attributive adjunct).
 Those weak students are to have tutors.
 Those pictures were sold in the painting exhibition.
d. Subjek dengan komponen nominal dari predikat.
 Students have got their own presents.
 Children looked like wild tribes.
e. Noun atau pronoun yang mengacu pada orang atau binatang dengan noun
lain yang mengacu pada sesuatu orang atau binatang tersebut.
 Applicants (they) have sent their letters of application.
 Respondents completed their questionnaires.
f. Ada beberapa kata benda tunggal yang mengacu kepada sekelompok individu
mempunyai kata kerja jamak jika titik berat perhatian kita para perorangannya dan
mempunyai pasangan kata kerja tunggal bila titik berat perhatian kita pada kelompok
sebagai satu kesatuan.
 The crowd were deeply affected.
 An immense crowd was assembled.
 My family are early risers.
 His family was on old one.
g. Kata benda yang mengacu kepada grup walau tidak terasa jelas perorangannya
menggunakan kata kerja jamak atau kadang kata kerja tunggal.
 The government are determined to resist aggression.
 The government has made a grant of land to the zoological society.
h. Collective nouns (kata benda kolektif) seperti cattle, clergy, police, people, horse
(=cavalry), foot (=infantry), poultry, vermin, hampir selalu berpasangan dengan kata
kerja jamak.
 Cattle were allowed to graze in the greenland.
 The police were on their duty.
 The poultry are being fed.
 People are always so impressive.

~ 61 ~
i. Indefinite pronouns seperti everybody, everyone, nobody, no one, anybody,
anyone berpasangan dengan kata kerja tunggal, namun bisa mengacu pada
kata ganti orang jamak.
 Everybody is jogging in the central park.
 No one has failed in their duty.
 Everyone brings their books to school.
 Nobody knows the answer.

3. Concord of Gender
Persesuaian dalam gender hanya seputar persesuaian antara subjek dengan
pronounnya. Male (laki-laki/jantan) mengacu kepada he-him-his dan
female (perempuan/betina) mengacu kepada she-her-hers dan kata benda
neutral (netral) yang mengacu kepada It-it-its. Kata benda netral mengacu
kepada kata benda atau orang yang tidak dapat diketahui jenis kelaminnya.

Male (He) Female (She) Neutral (It)


Mary, host, master Tom, hostess, mistress doctor, teacher
prince, king princess, queen baby, child
father, uncle mother, aunt an apple, a banana
cat, dog cow, ship student, pupil

Contoh:
 Mary met her friend in the Central Park.
 The Prince invited his great guests from the other kingdooms.
 The baby is keepin its toys.

4. Concord of Tense
Concord of tense biasanya terjadi antara finite verb dalam main clause
dengan finite verb dalam sub-clause dalam kalimat majemuk terutama
dalam indirect speech atau persesuaian tense dengan adverb of time.
a. Verb and verb
 He says that he knows all about that problem.
 He said that he knew all about it.

~ 62 ~
b. Verb and adverb of time
 We studied Mandarin many years ago.
 We study French every week.
 We are studying English now.

Latihan Pilihan Ganda


1. Everyone … where he actually comes from.
A. starts questioning
B. starting questioning
C. start question
D. start questioning
2. Barbara will visit her grandmother … by the end of this month.
A. by himself
B. by themselves
C. by herself
D. by myself
3. My electricity bill for this month ... very expensive.
A. are
B. is
C. were
D. was
4. What Ahmad said to me … me realize that I am wrong.
A. make
B. made
C. making
D. makes
5. Some professors start … experiment about herb medicine.
A. our
B. his
C. her
D. their

~ 63 ~
6. All the doctors in our hospital have their own practice room. Each of them … a
nurse to assist him.
A. need
B. will need
C. needs
D. needed
7. The Amazon valley is extremely important to the ecology of the earth. Forty
percent of the world’s oxygen ... there.
A. was produced
B. is produced
C. were produced
D. are produced
8. X: ‘What makes you so sad with the current reform movement in Indonesia?’
Y: ‘… bad effect on Indonesia’s economy.’
A. Its
B. It
C. their
D. they
9. The local radio newspapers as well as newspapers ... that riots have spread to
other places in the city.
A. is confirmed
B. confirm
C. are confirmed
D. confirms
10. The number of people attending the concert … not as had been expected.
A. were
B. being
C. was
D. to be

~ 64 ~
Latihan Melengkapi
1. Neither the students nor the professor … (be) happy about the result of the
experiment.
2. The installation on various high-tech electrical devices in our office …
(require) a sophisticated safety system.
3. The majority of the students … (agree) that the class today must be canceled.
4. None of participants … (be) allowed to come home early.
5. We did not buy these stuffs because … (they) price is so expensive.
6. The burden of economic problems … (frustrate) the majority of Indonesian
people.
7. The number of unemployed person … (be) increasing every year.
8. A number of employees … (be) on strike.
9. The woman across the street shouted my name and waved … (she) hand.
10. Different interpretations on the same event by various newspapers … (make)
readers confused and angry.

Latihan Error Recognition

1. It was a miracle that neither the passengers nor the driver were
A B C
seriously injured in the accident.
2. At the beginning of this semester, each of the students are given a
A B C
new time- table.
3. The number of illiterate people in our country have decreased
A B
drastically.
C
4. Each girl must contribute his share of the expenses.
A B C

~ 65 ~
5. As the production of these commodities greatly depend on the
A
availability of raw materials, they are very expensive.
B C
6. X: ‘Have you checked where most of the seminar participants comes
from?’ A
Y: ‘Oh yes, two-thirds of them come from various parts of Java.’
B C
7. A: ‘I have heard that the school where you teach is very good,’
B: ‘Oh yes, many of their graduates have become popular leaders of the
country.’ A B C
8. Sarah, Mario, and Clark have made up their minds for continuing
A
their study in Europe. No one can stop theirs.
B C
9. The President, accompanied by some of his ministers, are visiting the rural
village now. A B C
10. The way he smiles and talks always attract many girls.
A B C

~ 66 ~
Unit 9 MODALS
TEORI SINGKAT
Modals adalah kata kerja bantu (auxiliary) yang memberikan arti tambahan
dalam kalimat dan tidak mengubah bentuk gramatika suatu kata kerja.
1. CAN
a. Menyatakan kemampuan (ability).
 He can speak five languages.
 She can lift the heavy box.
b. Menyatakan kemungkinan (possibility).
 It can snow in April.
 It can rain in October.
c. Menyatakan ijin/kebolehan (permission).
 You can go now.
 She can leave soon.
d. Can = be able to jika digunakan dalam future/perfect tense.
 She has been able to solve the problem herself.
 They will be able to climb that mountain.
2. MAY
a. Menyatakan kebolehan/ijin (permission).
 May I ask you a question?
 May I come to your house?
b. Menyatakan kemungkinan (possibility).
 It may rain tonight.
 You are here now, she may stay at home alone.
c. Menyatakan harapan (wish).
 May you soon be well again.
 May God bless you.
3. WILL
a. Menyatakan future (to be going to).
 He will take an English course in Australia next semester.

~ 67 ~
 She is going to go to California tomorrow.
b. Menyatakan persetujuan dan keinginan.
 I will do whatever you think best.
 We will come if we have more time.
c. Menyatakan permintaan
 Will you close the window, please?
 Will you help me for a minute, please?
4. SHALL
a. Menyatakan future (to be going to) sebaiknya digunakan untuk subjek I
dan We.
 I shall leave for Harvard tomorrow.
 We shall go next.
b. Menyatakan persetujuan atau bantuan untuk melakukan sesuatu bagi orang
lain.
 Shall I answer the telephone for you (= Do you want me to answer the
telephone?)
 Shall I open the front door for you (=Do you want me to open the
front door for you?)
c. Menyatakan keputusan (decision) yang harus dilaksanakan.
 You shall go out now.
 I shall do the job right now.
5. MUST
a. Menyatakan kebutuhan (necessity) dan keharusan (=have to).
 I must take a rest. (=I have to take a rest.)
 My father must take the medical care in the city hospital.
b. Menyatakan kemungkinan kuat (strong possibility).
 She was absent yesterday. She must have been sick again.
 His cigar was still burning but he was not there.
He must have gone out.
c. Menyatakan kesimpulan (conclusion).
 The door is locked. He must not be at home.
 The window is open. The owner of the house must be in.

~ 68 ~
d. Must = to be to = am/is/are/have/has/was/were/had to
 She must enroll last week, it is stopped now. = She was to enroll last
week, it is stopped now.
 They must submit their assignments a week ago. = They were to
submit their assignments a week ago.
6. COULD
Bentuk lampau (past tense) dari CAN dan menyatakan kebolehan.
 You could come to my house yesterday night.
 Could I run the business myself?
7. MIGHT
Bentuk lampau (past tense) dari MAY dan menyatakan kemungkinan besar.
 If you invited him, he might come to the meeting.
 They might have left early because they arrived first and on time.
8. WOULD
Bentuk lampau (past tense) dari WILL dan menyatakan permintaan yang
lebih sopan.
 I would go if it did not rain yesterday.
 Would you like to come to my house tonight?
9. SHOULD
a. Menyatakan anjuran (advisability).
 You look pale, you should see a doctor.
 He seems sleepy when he drives. You should ask him to stop.
b. Menyatakan kewajiban (obligatory/duty) = ought to.
 Everyone should (ought to) respect parents.
 He should come to take the test on time.
c. Menyatakan kesimpulan yang sudah jelas.
 She took singing lesson for years, she should be an excellent singer.
 You lived in London for five years, you should speak English fluently.
10. OUGHT TO
Menyatakan keharusan yang tak terlaksanakan (seyogyanya/sebaiknya).
 You ought to have gone yesterday.(=You did not go yesterday.)
 He ought to have come last night. (=He did not come last night.)

~ 69 ~
Latihan Pilihan Ganda
1. Your glass is empty. … I get you some more fresh water?
A. Shall
B. Will
C. Should
D. Would
2. You are sick, but you haven’t gone to hospital to see a doctor. You … visit him
soon.
A. would
B. could
C. should
D. might
3. X: “I really don’t know why Josh didn’t come to the meeting”.
Y: “There … something wrong with him.
A. should have been
B. must have been
C. will have been
D. would have been
4. Yanti looks so pale and weak. He must be sick. The underlined sentence
means:
A. He has to be sick
B. Maybe he is sick
C. He is rather sick
D. I conclude that he is sick
5. “I could have somebody else to carry the heavy bags” It means:
A. I carried the heavy bags
B Somebody else carried the heavy bags for me
C. I would carry the heavy bags myself.
D. I didn’t want to carry the heavy bags.

~ 70 ~
6. The police officer warned the crowd that he … order shooting if they created
trouble.
A. could
B. should
C. might
D. would
7. There was a time when I … stay up late, but these days I go to bed early.
A. used to
B. ought to
C. should
D. would
8. Tia : I’m writing a composition, and there is a word that I don’t
know how to spell.
Wahyu : You … open the dictionary.
A. will
B. may
C. should
D. shall
9. Tom told me a lot about the Philippines. He … there for a long time.
A. must have lived
B. might be living
C. ought to have lived
D. should be living
We are going on a long trip, so we ... have the car checked.
A. will
B. must
C. may
D. can

~ 71 ~
Latihan Melengkapi
1. My father is very smart. He … speak five languages.
2. … you show me where the post office is?
3. Look! The girl is falling into the river. We … rescue her now.
4. John is absent today. He … be absent.
5. You … (not) touch the wall. My father has just painted it.
6. … you come to my house tonight?
7. If you don’t bring your dictionary, you … borrow mine now.
8. I … play a guitar when I was 10 years old.
9. Sorry, I am very busy now. I … (not) help you.
10. Tom failed in his exams. I think he … harder.

Latihan Error Recognition


1. We trimmed the hedges so that more sunlight can get through.
A B C
2. You will better tone up your muscle if you want to try out for
A B C
gymnastics this year.

3. Every time I lost my tooth, my grandfather will give me 25 cents.


A B C
4. Angela lives in Minnesota now, she uses to live in Alabama.
A B C
5. Would they make you cut your hair if you join the Air Force?
A B C
6. I will have fallen asleep early last night. I don’t remember anything
A B C
after 10.00.
7. It’s going to be warm in the conference room. You must to take off
A B C
your coat.
8. You mustn’t throw litter from your vehicle because it should pose a
A B C
risk to the environment.
9. We must study hard, because next April we would have the National
A B C
Examination.
10. I don’t have enough money to buy lunch. Should you lend me a
A B C
couple of dollars?

~ 72 ~
Unit 10 PASSIVE VOICE

TEORI SINGKAT
Kalimat pasif adalah kalimat yang subyeknya dikenai pekerjaan atau
tindakan atau perbuatan.. Subyek dalam kalimat pasif berfungsi sebagai receiver
atau penderita sedangkan unsur obyeknya sebagai agent/doer atau pelaku. Tense
dalam kalimat pasif mengikuti Tense dalam kalimat aktif.
Kalimat pasif memiliki pola umum sebagai berikut:

To be + Verb-3 (past participle)

Beberapa formulasi “Passive Voice”:

1. Present Simple & Past Simple:


am  A great deal of property is destroyed by
is hurricanes every year.
are Makna: Selalu/sering dirusak.
+ Verb-3
 Fifty houses was destroyed by the tornado last
was year.
were Makna: Sudah dirusak

2. Present Continuous & Past Continuous:


am  Several new proposals are being considered by
is the committee.
are + being + Makna: Saat ini sedang dipertimbangkan
Verb-3  Several new proposals were being considered by
Was the committee.
were Makna: Dulu sedang dipertimbangkan

3. Present Perfect & Past Perfect:


 Some new equipment has been ordered by the
Has company.
Have + been + Makna: telah/baru saja dipesan
Had Verb-3  Some new equipment had been ordered by the
company.
Makna: Dulu telah dipesan

~ 73 ~
4. Modals
be +
 Your homeworks must be done.
Verb-3
Makna: harus dikerjakan
Modals
Have +  A new bridge will be built by the government.
been + Makna: akan dibangun
Verb-3

5. Passive Gerund

 Active: I remember my father taking me to the


beach.
Makna: membawa
Being + Verb-3
 Passive: I remember being taken to the beach by
my father.
 Makna: dibawa

Catatan:

1. Kata kerja-kata kerja berikut ini dapat dibuat bentuk pasif dalam konstruksi
kalimat khusus:
Acknowledge, assume, believe, claime, consider, estimate, feel, find, know,
presume, report, say, think, understand.
 It was reported that the robber was arrested by the policemen.
 It was said that he was jealous of her.
2. Kata kerja-kata kerja berikut ini dapat dibuat bentuk pasif dalam konstruksi
kalimat khusus:
Think, know, believe, understand, report, say, suppose
 He is thought to be living abroad.
 That man is believed as their hero in that village.

~ 74 ~
TRANSFORMASI “ACTIVE VOICE” “PASSIVE VOICE”
Tense Active Voice Passive Voice
Somebody cleans this This room is cleaned.
Present Simple
room.
Somebody cleaned this This room was cleaned.
Past Simple
room.
Present Somebody is cleaning this This room is being cleaned.
Continuous room.
Somebody was cleaning This room was being
Past Continuous
this room. cleaned..
Somebody has cleaned this This room has been
Present Perfect
room. cleaned.
Somebody had cleaned this This room had been
Past Perfect
room. cleaned.

I. Latihan Pilihan Ganda


1. On my way home, I saw a dog ... by a boy.
A. beating
B. being beaten
C. it was beaten
D. be beaten
2. Fred was offered the choice between two places. It means:
.… the choice between to places.
A. Fred offered someone
B. Fred had offered
C. Someone offered Fred
D. They would offer Fred
3. The production process in the car company ….
A. needs to be monitored
B. are needed to monitor
C. they need to monitor

~ 75 ~
D. are needed to be monitored
4. “Some workers were loading the containers into the van”. The passive form of
the above sentence is:
“The containers … into the truck.
A. were loaded
B. are loading
C. be loaded
D. were being loaded
5. She became quite nervous when she knew that her speech ….
A. was to record
B. being recorded
C. to be recording
D. was being recorded
6. The school has a lot of classrooms; each of them … equipped with an overhead
projector.
A. are
B. is
C. being
D. to be
7. “Which painting will be exhibited tomorrow?”
“We don’t know yet; they … by a team.
A. are still being selected
B. are still selecting
C. still be selected
D. still selecting
8. “Last night someone broke into our house.”
“Oh, dear! …?”
A. Did anything take
B. Was anything taken
C. Anything to be taken
D. Was anything being taken

~ 76 ~
9. “Can we get tickets for the second show of the Spiderman III?”
“I think they .…”
A. are sold
B. sold out
C. are selling them out
D. were sold
10. The picture is not here anymore; it must have ....
A. being taken away
B. taken away
C. been taking away
D. been taken away

II. Latihan Melengkapi

Change the active sentences below into passive voice.


1. The servant cleans the room every day.
2. The students are using the computers now.
3. Ahmad has taken an English course.
4. My window was broken. My father fixed it yesterday.
5. They were watching the television when their mother came.
6. The police had not found your car yet.
7. Jane will buy some novels because she has a literary assignment.
8. The teacher should make the lesson more interesting.
9. We must write the answers on one side of the paper.
10. A plumber is going to fix the leaky faucet.

III. Latihan Error Recognition


1. Calcium are known as a nutrient that will decrease hypertension and colon
cancer. A B C

2. A body of volunteers have been organized to aid the helpless in their


A B C
struggle for survival.

~ 77 ~
3. The report cites that the number of employees in the private sector
A B
have doubled between 2007 and 2010.
C

4. A number of visitors of the zoo is warned not to feed the animals for
A B C
several health reasons.

5. Tika : How often are paychecks issued?


Lia : Paychecks will give out every two weeks.
A
Tika : Good. I think that’s better than once a month.
B C
6. I am confusing because I cannot get what the teacher says.
A B C
7. The anchors who have nice appearance on TV will be the winner; this
A B
based on the viewers’ polling.
C
8. X: What happened to your car?
A
Y: It were hit by a truck.
B C
X: So sorry to hear that.
9. The missing girl found walking barefoot along the beach yesterday.
A B C
10. A: Will we be able to see the animals are fed while we’re here at the zoo?
A B
B: Yes, certainly. They will feed at five o’clock today.
C

~ 78 ~
REFERENSI

Azar, B. Schramper. 1989. Understanding and Using English Grammar.


Englewood Cliffs. NJ07632. Prentice-Hall Regents. Prentice-Hall Press.
Berry, T.E. 1978. The Most Grammar Mistakes in English Usage. New Delhi:
Tata McGraw-Hill Publishing Company. Ltd.
Fitikides, T.J. 1990. Common Mistakes in English. Fifth Edition. London:
Longman Group Company.
Frank, M. 1972. Modern English: A Practical Reference Guide. USA: Prentice-
Hall. Inc.
Gatherer, W.A. 1986. The Student’s Handbook of Modern English. Jakarta:
Penerbit PT. Gramedia Pustaka.
Hayden, R.E. 1990. Mastering American English. Englewood Cliffs. New Jersey:
Prentice-Hall. Inc.
Hartono, R. 2007. A Communicative Grammar. Surakarta: UNS Press.
Hornby, A.S. 1988. Guide to Patterns and Ussage in English. Second Edition.
Oxford: Oxford University Press.
Hornby, A.S. 1986. Oxford Advance Learner’s Dictionary of Current English.
Oxford: Oxford University Press.
Lado. M.J. 1996. Common Errors in English. Jakarta: CV. Tulus Jaya
Martinet, A.V. & Thomson, A.J. 1986. A Practical English Grammar. Oxford:
Oxford University Press.
Murcia, M.C. & Freeman, D.L. 1983. The Grammar Book: An ESL/EFL
Teacher’s Guide Course. Rowley, Massachusetts 01969: Newbury House
Publishers. Inc.
Smedley, D. 1983. Teaching the Basic Skills: Spelling, Punctuation and Grammar
in Secondary English. USA: Methuen & Co.

~ 79 ~
KUNCI JAWABAN
UNIT 1

I. Latihan Pilihan Ganda

1. C
2. D
3. B
4. C
5. A
6. B
7. B
8. B
9. A
10. D

II. Latihan Melengkapi

1. have
2. had studied
3. was washing
4. will watch
5. will have taken
6. is running
7. has been cooking
8. Have you completed
9. will not attend
10. bought

III. Latihan Error Recognition

1. B, seharusnya works karena my sister adalah orang ketiga tunggal.


2. B, seharusnya has karena recently menggunakan simple present tense.
3. B, seharusnya stole karena ada kata ran, dalam bentuk simple past tense.
4. C, seharusnya will have arrived karena adanya kata keterangan by the end
of this week.
5. B, seharusnya will be karena mengungkapkan sesuatu yang belum terjadi.
6. C, seharusnya comes karena digunakan dalam dependent clause.
7. A, seharusnya require karena peas adalah kata benda jamak.
8. A, seharusnya moves karena mengungkapkan suatu general truth.
9. A, seharusnya fixes karena mengungkapkan suatu general truth.
10. B, seharusnya went karena mengungkapkan sesuatu yang terjadi di masa
lalu.

~ 80 ~
UNIT 2

I. Latihan Pilihan Ganda

1. B
2. C
3. A
4. D
5. C
6. B
7. A
8. D
9. B
10. C

II. Latihan Melengkapi

1. her; objective pronoun me-refer ke Anita.


2. She, it; she subjective pronoun me-refer ke Anita, it objective pronoun me-
refer ke a cat.
3. mine, it; mine independent possessive pronoun me-refer keI, it objective
pronoun me-refer ke pen.
4. himself; reflexive pronoun me-refer ke Anton.
5. everything; mengungkapkan indefinite pronoun.
6. Those/These; demonstrative pronoun untuk kata benda jamak books.
7. his; dependent possessive pronoun me-refer kesomebody.
8. ourselves; reflexive pronoun me-refer ke we.
9. anybody/anyone; mengungkapkan indefinite pronoun.
10. this; demonstrative pronoun untuk kata benda tunggal song.

III. Latihan Error Recognition

1. C, seharusnya our sebagai dependent possessive pronoun.


2. B, seharusnya yours sebagai independent possessive pronoun.
3. B, seharusnya He sebagai subjective pronoun me-refer ke my father.
4. A, seharusnya yourself sebagai reflexive pronoun me-refer ke you.
5. B, seharusnya them karena objective pronoun me-refer ke the students.
6. C, seharusnya myself sebagai reflexive pronoun me-refer ke I.
7. A, seharusnya These karena demonstrative pronoun jamak me-refer ke
sandals.
8. C, seharusnyathosekarena demonstrative pronoun jamak me-refer
keproducts.
9. C, seharusnya themselves sebagai reflexive pronoun me-refer ke they.
10. A, seharusnya anyone/anybody sebagai indefinite pronoun dalam kalimat
tanya.

~ 81 ~
UNIT 3

I. Latihan Pilihan Ganda

1. B
2. A
3. D
4. D
5. C
6. B
7. C
8. C
9. A
10. B

II. Latihan Melengkapi

1. because/since
2. either… or…
3. Yet… still…/But… still…
4. consequently/therefore
5. unless
6. not only… but also…/both… and…
7. Although/Even though
8. in case
9. Neither… nor…
10. whether or not

III. Latihan Error Recognition

1. C, seharusnyain case untuk menyatakan ‘jika seandainya’.


2. B, seharusnya however/nevertheless/nonetheless untuk menyatakan
‘tetapi/namun’.
3. A, seharusnya because/since untuk menyatakan sebab akibat.
4. B, seharusnya unless untuk menyatakan ‘jika tidak’.
5. A, seharusnya neither karena berpasangan dengan ‘nor’.
6. A, seharusnya unless untuk menyatakan ‘jika tidak’.
7. B, seharusnya consequently/therefore untuk menyatakan ‘oleh karena itu’.
8. C, seharusnya but also karena berpasangan dengan ‘not only’.
9. B, seharusnya although/even though untuk menyatakan ‘walaupun’.
10. A, seharusnya or karena berpasangan dengan ‘either’.

~ 82 ~
UNIT 4

I. Latihan Pilihan Ganda

1. D
2. B
3. C
4. D
5. C
6. D
7. B
8. B
9. A
10. B

II. Latihan Melengkapi

1. will submit; Conditional Sentence Type I.


2. would tell; Conditional Sentence Type II.
3. would have been, Conditional Sentence Type III.
4. had; Conditional Sentence Type II.
5. needed; Conditional Sentence Type II.
6. would have found; Conditional Sentence Type III.
7. clean; Conditional Sentence Type I.
8. were; Conditional Sentence Type II.
9. watches; Conditional Sentence Type I.
10. explained; Conditional Sentence Type II.

III. Latihan Error Recognition

1. C, seharusnya will buy karena Conditional Sentence Type I.


2. A, seharusnya were karena Conditional Sentence Type II.
3. C, seharusnya had studied karena Conditional Sentence Type III.
4. B, seharusnya would be karena Conditional Sentence Type II.
5. A, seharusnya would have sent karena Conditional Sentence Type III.
6. B, seharusnya gives karena Conditional Sentence Type I.
7. B, seharusnya would apply karena Conditional Sentence Type III.
8. A, seharusnya will add karena Conditional Sentence Type I.
9. B, seharusnya hadn’t been karena Conditional Sentence Type III.
10. B, seharusnya will go karena Conditional Sentence Type I.

~ 83 ~
UNIT 5

I. Latihan Pilihan Ganda

1. C
2. B
3. A
4. C
5. A
6. A
7. B
8. D
9. B
10. C

II. Latihan Melengkapi

1. their teacher does too/so does their teacher.


2. he doesn’t either/neither does he.
3. my uncle did to/so did my uncle.
4. you can’t either/neither can you.
5. I won’t either/neither will I.
6. so was.
7. won’t.
8. we do too/so do we.
9. don’t.
10. the girls are too/ so are the girls.

III. Latihan Error Recognition

1. A, seharusnya will start.


2. C, seharusnya is.
3. B, seharusnya play.
4. C, seharusnya either.
5. A, seharusnya gave.
6. C, seharusnya have.
7. B, seharusnya cannot.
8. A, seharusnya was.
9. B, seharusnya will study.
10. C, seharusnya neither.

~ 84 ~
UNIT 6

I. Latihan Pilihan Ganda

1. C
2. D
3. B
4. C
5. D
6. A
7. C
8. A
9. B
10. D

II. Latihan Melengkapi

1. younger; comparative degree karena ada kata ‘than’.


2. biggest; superlative degree karena ada kata ‘the’.
3. smaller; comparative degree karena ada kata ‘than’.
4. long; positive degree karena ada kata ‘as…as’.
5. faster; comparative degree karena ada kata ‘than’.
6. most difficult; superlative degree karena ada kata ‘the’.
7. more beautiful; comparative degree.
8. most diligent; superlative degree karena ada kata ‘the’.
9. strong; positive degree karenaada kata ‘as…as’.
10. cleanest; superlative degree karenaada kata ‘the’.

III. Latihan Error Recognition

1. B, seharusnya less.
2. C, seharusnya fluent.
3. A, seharusnya best.
4. B, seharusnya more expensive.
5. B, seharusnya most useful.
6. B, seharusnya more.
7. C, seharusnya sharp.
8. A, seharusnya hardest.
9. B, seharusnya harder.
10. C, seharusnya most badly.

~ 85 ~
UNIT 7

I. Latihan Pilihan Ganda

1. C
2. D
3. B
4. B
5. A
6. C
7. A
8. B
9. B
10. D

II. Latihan Melengkapi

1. communities
2. deer
3. thieves
4. Oxen, geese
5. phenomena
6. Department
7. recovery
8. association
9. entry
10. happiness

III. Latihan Error Recognition

1. A, seharusnya plans.
2. B, seharusnya words.
3. B, seharusnya work karena work adalah uncountable noun.
4. A, seharusnya children.
5. C, seharusnya happiness karena happy adalah kata sifat (adjective).
6. A, seharusnya them sebagai objective pronoun.
7. C, seharusnya application karena apply adalah kata kerja (verb).
8. A, seharusnya marketing karena market adalah kata kerja (verb).
9. B, seharusnya announcement karena announce adalah kata kerja (verb).
10. C, seharusnya gifts karena give adalah kata kerja (verb).

~ 86 ~
UNIT 8

I. Latihan Pilihan Ganda

1. A 6. C
2. C 7. B
3. B 8. B
4. D 9. B
5. D 10. C

II. Latihan Melengkapi

1. is; menyesuaikan subyek yang paling dekat, yaitu ‘the professor’.


2. requires; menyesuaikan subyek yang bersifat tunggal, yaitu‘The
installation…’.
3. agree; menyesuaikan subyek yang bersifat jamak, yaitu ‘The majority of
students’
4. is; menyesuaikan subyek yang bersifat tunggal, yaitu ‘None of…’.
5. their; menyesuaikan obyek yang bersifat jamak, yaitu ‘these stuffs’.
6. frustrates; menyesuaikan subyek yang bersifat tunggal, yaitu ‘The
burden…’.
7. is; menyesuaikan subyek yang bersifat tunggal, yaitu ‘ The number of…’.
8. are; menyesuaikan subyek yang bersifat jamak, yaitu ‘A number of…’.
9. her; menyesuaikan subyek yang di-refer, yaitu ‘The woman’.
10. make; menyesuaikan subyek yang bersifat jamak, yaitu ‘Different
interpretations’.

III. Latihan Error Recognition

1. C, seharusnya was karena menyesuaikan subyek yang paling dekat, yaitu


‘the driver’.
2. C, seharusnya is karena menyesuaikan subyek yang bersifat tunggal, yaitu
‘each’.
3. B, seharusnya has karena menyesuaikan subyek yang bersifat tunggal,
yaitu ‘ The number of…’.
4. B, seharusnya her karena menyesuaikan subyek ‘Each girl’.
5. A, seharusnya depends karena menyesuaikan subyek yang bersifat
tunggal, yaitu ‘the production of…’.
6. A, seharusnyacomekarenamenyesuaikansubyek yang bersifatjamakyaitu
‘most of…’.
7. B, seharusnya its karena menyesuaikan subyek yang bersifat tunggal, yaitu
‘the school’.
8. C, seharusnyathemkarenamenyesuaikan objective pronoun untuksubyek
‘they’.
9. C, seharusnyaiskarenamenyesuaikansubyek yang bersifattunggalyaitu ‘The
President’.
10. B, seharusnyaattractskarenamenyesuaikansubyek yang
bersifattunggalyaitu ‘The way’.
~ 87 ~
UNIT 9

I. Latihan Pilihan Ganda

1. A
2. C
3. B
4. D
5. B
6. A
7. A
8. C
9. A
10. B

II. Latihan Melengkapi

1. can; untuk mengungkapkan kemampuan (ability).


2. Could/Can; could digunakan untuk megungkapkan permintaan yang lebih
sopan.
3. must; untuk mengungkapkan keharusan.
4. may/might/must; untuk mengungkapkan prediksi, tergantung besarnya
keyakinan kita akan hal tersebut.
5. must not, untuk mengungkapkan larangan (prohibition).
6. Could/Can/Will, untuk mengungkapkan kemungkinan.
7. may/can; untuk mengungkapkan kebolehan.
8. used to; untuk mengungkapkan kebiasaan yang dilakukan di masa lalu.
9. cannot; untuk mengungkapkan ketidakmampuan.
10. must/should study; untuk mengungkapkan keharusan/saran kuat.

III. Latihan Error Recognition

1. C, seharusnya could karena menyesuaikan kata kerja lampau ‘trimmed’.


2. A, seharusnya would better.
3. C, seharusnya would karena menyesuaikan kata kerja lampau ‘lost’.
4. B, seharusnya used to.
5. A, seharusnya Will karena menyesuaikan kata kerja present ‘join’.
6. A, seharusnya must karena mengungkapkan keyakinan yang kuat.
7. C, seharusnyamust karena modal tidak bisadi ikuti kata ‘to’.
8. B, seharusnya can untuk menyatakan kemungkinan yang kuat.
9. B, seharusnya will karena menyesuaikan kata keterangan ‘next April’.
10. C, seharusnya Could/Can, could digunakan untuk megungkapkan
permintaan yang lebih sopan.

~ 88 ~
UNIT 10

I. Latihan Pilihan Ganda

1. B
2. C
3. A
4. D
5. D
6. B
7. A
8. B
9. A
10. D

II. Latihan Melengkapi

1. The room is cleaned by the servant every day.


2. The computers are being used by the students now.
3. An English course has been taken by Ahmad.
4. My window was fixed by my father yesterday.
5. The television was being watched by them when their mother came.
6. Your car had not been found by the police.
7. Some novels will be bought by Jane because she has a literary assignment.
8. The lesson should be made more interesting by the teacher.
9. The answers must be written by us on one side of the paper.
10. The leaky faucet is going to be fixed by the plumber.

III. Latihan Error Recognition

1. A, seharusnya is known karena menyesuaikan uncountable noun ‘calcium’.


2. A, seharusnya has been organized karena menyesuaikan kata benda
tunggal ‘A body’.
3. C, seharusnya has doubled karena menyesuaikan kata benda tunggal ‘the
number of…’.
4. B, seharusnya are warned karena menyesuaikan kata benda jamak ‘A
number of…’.
5. A, seharusnya will be given.
6. A, seharusnya am confused.
7. C, seharusnya is based.
8. B, seharusnya was hit karena menyesuaikan kata benda tunggal ‘It’.
9. B, seharusnya was found.
10. C, seharusnya will be fed.

~ 89 ~
-'

English Grammar
for Indonesian Students

ISBN C?A-tfle-?33r{5-h-q

lllrjrlrjll|l|ilL|ffi[[Lil

Anda mungkin juga menyukai